You are on page 1of 397

MATH 360: Theory of Investment and Credit

Albert Cohen
Actuarial Sciences Program
Department of Mathematics
Department of Statistics and Probability
C336 Wells Hall
Michigan State University
East Lansing MI
48823
albert@math.msu.edu
acohen@stt.msu.edu

Albert Cohen (MSU)

MATH 360: Theory of Investment and Credit

MSU Spring 2014

1 / 223

Logistics

All course materials available on class page (www.math.msu.edu)


Syllabus too
Course textbook is S.A. Brovermans Mathematics of Investment
and Credit, 5th edition or later
Some questions on these slides, and on in class exam preparation
slides, are taken from the third edition of Brovermans book. Please
note that Actex owns the copyright for that material. No portion of
the ACTEX textbook material may be reproduced in any part or by
any means without the permission of the publisher. We are very
thankful to the publisher for allowing posting of these notes on our
class website.
Supplementary book is Finan, available online

Albert Cohen (MSU)

MATH 360: Theory of Investment and Credit

MSU Spring 2014

2 / 223

Introduction

The value of money is a function of the time that passes while it is stuffed
under a mattress, deposited in a bank account, or invested in an asset.
Just what that function is depends on many circumstances, and we will
spend our time investigating many real-world examples

Albert Cohen (MSU)

MATH 360: Theory of Investment and Credit

MSU Spring 2014

3 / 223

Main Tools

Calculus, and the study of Geometric Series, provides

Albert Cohen (MSU)

MATH 360: Theory of Investment and Credit

MSU Spring 2014

4 / 223

Main Tools

Calculus, and the study of Geometric Series, provides


the machinery necessary for solving for the final value of an
investment
the optimal time to switch accounts
the interest rate needed to plan fixed income investments
among other wonderful things

Albert Cohen (MSU)

MATH 360: Theory of Investment and Credit

MSU Spring 2014

4 / 223

Main Tools

Calculus (1-d only!)

Albert Cohen (MSU)

MATH 360: Theory of Investment and Credit

MSU Spring 2014

5 / 223

Main Tools

Calculus (1-d only!)


Can you take a derivative?
Can you find the maximum of a function?

Albert Cohen (MSU)

MATH 360: Theory of Investment and Credit

MSU Spring 2014

5 / 223

Main Tools

Calculus (1-d only!)


Can you take a derivative?
Can you find the maximum of a function?
Sequences and Series
Do you know what a geometric series is? What happens as n ?

Albert Cohen (MSU)

MATH 360: Theory of Investment and Credit

MSU Spring 2014

5 / 223

Main Tools

Calculus (1-d only!)


Can you take a derivative?
Can you find the maximum of a function?
Sequences and Series
Do you know what a geometric series is? What happens as n ?
n
X
k=0

Albert Cohen (MSU)

k =

1 n+1
1

MATH 360: Theory of Investment and Credit

(1)

MSU Spring 2014

5 / 223

Interest Accumulation and Effective Rates of Interest (Ex


1.1)

Begin by investing 1000 at 9% per annum. Compound annually for 3


years. End up with...

Albert Cohen (MSU)

MATH 360: Theory of Investment and Credit

MSU Spring 2014

6 / 223

Interest Compounded Monthly

Sometimes, interest rate is quoted per annum, but compounded monthly.


For previous example, assume again 9% but compounded monthly. Then
"
FV = 1000

0.09
1+
12

12 #3
(2)

= 1000 (1.0938)3

Albert Cohen (MSU)

MATH 360: Theory of Investment and Credit

MSU Spring 2014

7 / 223

Interest Compounded Monthly

Sometimes, interest rate is quoted per annum, but compounded monthly.


For previous example, assume again 9% but compounded monthly. Then
"
FV = 1000

0.09
1+
12

12 #3
(2)

= 1000 (1.0938)3
In other words, the equivalent or effective annual rate is 9.38%

Albert Cohen (MSU)

MATH 360: Theory of Investment and Credit

MSU Spring 2014

7 / 223

Fluctuating Interest Rates

It may be that over a longer period of time, the rate of return on an initial
investment A(0) may fluctuate. At time n, the value of the investment is
now A(n).

Albert Cohen (MSU)

MATH 360: Theory of Investment and Credit

MSU Spring 2014

8 / 223

Fluctuating Interest Rates

Guess: A(n) = A(0) (1 + r )n


Reality: A(n) = A(0) (1 + r1 ) ... (1 + rn )
In a bank account or bond, the interest rate, ri is guaranteed to be small
but positive. For a general investment, there is no guarantee that an
investment will not shrink in value over a period of time.

Albert Cohen (MSU)

MATH 360: Theory of Investment and Credit

MSU Spring 2014

9 / 223

Arithmetic vs Geometric Means


n
. The
One other possible guess for an average rate of return is r = r1 +...r
n
question of ordering, ie which average rate is bigger, reduces to the
question:

r1 + ...rn n
> (1 + r1 ) ... (1 + rn )?
(3)
Is 1 +
n
To answer this, define for k {1, 2, .., n},

xk = 1 + rk .

(4)

Then, our question is reframed as:


 x + ...x n
1
n
Is
> (x1 ) ... (xn )?
n

Albert Cohen (MSU)

MATH 360: Theory of Investment and Credit

(5)

MSU Spring 2014

10 / 223

Arithmetic vs Geometric Means

The answer is yes, and there are many proofs. One of them is by
induction. Another is to apply Jensens Inequality:
For
a concave function f , for example if f 00 (x) 0 for all x in our domain
P
real numbers ak such that nk=1 ak 6= 0,
real numbers xk ,
it follows that
f

Albert Cohen (MSU)

! P
Pn
n
ak xk
a f (x )
k=1
Pn
Pn k k .
k=1
k=1 ak
k=1 ak

MATH 360: Theory of Investment and Credit

(6)

MSU Spring 2014

11 / 223

Arithmetic vs Geometric Means

By Jensens Inequality, if we define f (x) = ln (x) and each ak = n1 , then


we retain
ln

n

 x + ... + x  1 X

1
n
ln (xk ) = ln n x1 ... xn

n
n

(7)

k=1

and so by taking exponentials of both sides we are done.


The inequality can be shown to be strict if the {xk }nk=1 are not all the
same. In other words, if even one interest rate is different between periods,
the the arithmetic average rate is strictly greater than the geometic rate.

Albert Cohen (MSU)

MATH 360: Theory of Investment and Credit

MSU Spring 2014

12 / 223

Example 1.2

Table: Average vs. Annual Rate of Return

Year
Annual Rate
Average Rate

2002
6.9%
6.9%

2001
6.4%
6.65%

2000
9.4%
7.56%

1999
3.0%
4.82%

1998
7.8%
5.41%

We can calculate the average interest rate iav via


(1 + iav )n = (1 + r1 ) (1 + r2 ) ... (1 + rn )

Albert Cohen (MSU)

MATH 360: Theory of Investment and Credit

MSU Spring 2014

(8)

13 / 223

Example 1.3

A very nice example is the following: On Jan 1, 2000, Smith deposits 1000
into an account with 5% annual interest. The interest is paid on every Dec
31. Smith withdraws 200 on Jan 1, 2002, deposits 100 on Jan 1 2003 and
again withdraws from the account on Jan 1 2005, this time 250. What is
the balance in the account just after the interest is paid on Dec 31, 2006?
Hint: Think of a deposit as a loan to the bank, a withdrawal as a loan to
Smith, and at the end of the term (7 years,) both bank and Smith settle
up accounts.

Albert Cohen (MSU)

MATH 360: Theory of Investment and Credit

MSU Spring 2014

14 / 223

Example 1.3
We can replicate the cash flows of this example by stating that Smith has
invested both positive and negative amounts of money over the period of
time:
Table: Example 1.3

Year
1000
200
100
250

Albert Cohen (MSU)

Period
7 years
5 years
4 years
2 years

MATH 360: Theory of Investment and Credit

MSU Spring 2014

15 / 223

Example 1.3
We can replicate the cash flows of this example by stating that Smith has
invested both positive and negative amounts of money over the period of
time:
Table: Example 1.3

Year
1000
200
100
250

Period
7 years
5 years
4 years
2 years

Hence,
A(7) = 1000 (1.05)7 + (200) (1.05)5
+ 100 (1.05)4 + (250) (1.05)2 = 997.77
Albert Cohen (MSU)

MATH 360: Theory of Investment and Credit

MSU Spring 2014

(9)
15 / 223

Example 1.3
Another way of computing A(7) is to do so recursively:
A(0) = 1000
A(1) = A(0) 1.05 = 1000 (1.05)
A(2) = A(1) (1.05) 200
= 1000 (1.05)2 200
A(3) = A(2) (1.05) + 100
= 1000 (1.05)3 200 (1.05) + 100

(10)

A(4) = A(3) (1.05)


A(5) = A(4) (1.05) 250
A(6) = A(5) (1.05)
A(7) = A(6) (1.05)
= 1000 (1.05)7 + (200) (1.05)5
+ 100 (1.05)4 + (250) (1.05)2 = 997.77
Albert Cohen (MSU)

MATH 360: Theory of Investment and Credit

MSU Spring 2014

16 / 223

Linear Interest

Another way interest can be designed to accrue is linearly Symbolically, if


A(0) is the initial value of our investment, then the final value is
Alinear (t) = A(0) (1 + i t)

(11)

Notice that at any time k t = n,


Acompound (n) = A(k) (1 + i)nk
= A(0) (1 + i)k (1 + i)nk
(12)

Alinear (n) = A(0) (1 + i n)


6= A(k) (1 + i (n k))
= A(0) (1 + i k) (1 + i (n k))

Albert Cohen (MSU)

MATH 360: Theory of Investment and Credit

MSU Spring 2014

17 / 223

Bonus Question

Assume you can select


from two interest rates j, i where 0 < i < j < 1 and
time t [0, 1]
where you can decide to switch from a bank paying j for a time of length t
to one paying i for the remaining time 1 t .
At what time t would this be optimal to do, if at all?

Albert Cohen (MSU)

MATH 360: Theory of Investment and Credit

MSU Spring 2014

18 / 223

Bonus Question
In general, if we switch at, then it must be that
1 + j < (1 + jt)(1 + (i(1 t))
1 + j < 1 + jt + i(1 t) + ijt(1 t)
(13)

(j i)(1 t) < ijt(1 t)


j i
< t.

ij
It follows that if 0 <

ji
ij

< 1, we are able to justify such a switch!

The inequality linking i to j is now


i <j <

Albert Cohen (MSU)

i
.
1i

MATH 360: Theory of Investment and Credit

(14)

MSU Spring 2014

19 / 223

Bonus Question
The optimal time to switch is when f (t) := (1 + jt)(1 + i(1 t)) is
maximized:
i
dh
(1 + jt)(1 + i(1 t))
dt
= (j i) + ij(1 2t).
1 1 j i
= +
.
2 2
ij

0 = f 0 (t) =

toptimal

Of course, toptimal < 1 as i < j <


bank account is

i
1i

and so the optimal value of the

 1 1 j i i
h
 1 1 j i ih
+
1+i

.
f (toptimal ) = 1 + j
2 2
ij
2 2
ij

Albert Cohen (MSU)

(15)

MATH 360: Theory of Investment and Credit

MSU Spring 2014

(16)

20 / 223

Accumulated Amount Function

A(t) is also known as the A.A.F., and


A (t2 ) = A (t1 ) (1 + it2 )
A (t2 ) A (t1 )
it2 :=
A (t1 )

(17)

it2 is known as the effective rate for our investment from t1 to t2 .

Albert Cohen (MSU)

MATH 360: Theory of Investment and Credit

MSU Spring 2014

21 / 223

Present Value

A dollar today is certainly more valuable than a dollar tomorrow, and even
more valuable than a dollar next year. To reflect this idea, we say that the
present value of a unit of currency one year from now is , where
1
.
(18)
1+i
is also known as the discount factor. This factor works as the inverse of
the interest gained on an investment of a unit of currency for one year.
For example, the present value of 25, 000 at a rate of 5% per annum, 25
25000
years from now, is (1.05)
25

Albert Cohen (MSU)

MATH 360: Theory of Investment and Credit

MSU Spring 2014

22 / 223

Equation of Value

In calculating the present value of an investment, we are working


backwards from a fixed outcome to calculate its value to us today.
But, there may be more than one payment expected in the future, such as
when planning for retirement. They may also be balanced by further
investment made in the future.

Albert Cohen (MSU)

MATH 360: Theory of Investment and Credit

MSU Spring 2014

23 / 223

Example 1.5

Loan Shark pays you 1000 every week, starting today. There are 4
payments in total, at 8% weekly interest. Starting the week after the last
1000, you are to repay the loan in 3 consecutive weekly installments of
1100 at 8% weekly interest, plus a fourth payment X at 8% weekly
interest. What is X ?

Albert Cohen (MSU)

MATH 360: Theory of Investment and Credit

MSU Spring 2014

24 / 223

Example 1.5

Table: Example 1.5: Payments In

Week
Payment in

0
1000

1
1000

2
1000

3
1000

Table: Example 1.5: Payments Out

Week
Payment out

Albert Cohen (MSU)

4
1100

5
1100

6
1100

MATH 360: Theory of Investment and Credit

7
X

MSU Spring 2014

25 / 223

Example 1.5

PV0 (in) = PV0 (out)

Albert Cohen (MSU)

MATH 360: Theory of Investment and Credit

MSU Spring 2014

26 / 223

Example 1.5

PV0 (in) = PV0 (out)


1000
1000
1000
+
PV0 (in) = 1000 +
+
2
1.08
1.08
1.083
1100
1100
1100
X
PV0 (out) =
+
+
+
4
5
6
1.08
1.08
1.08
1.087
X = 2273.79

Albert Cohen (MSU)

MATH 360: Theory of Investment and Credit

MSU Spring 2014

(19)

26 / 223

Nominal Interest Rates

When a credit card has a 24% nominal annual interest rate, you do not
only pay 24% on the balance. Rather, that 24% is broken down into 12
monthly interest charges on the average balance over a 30 day billing
cycle. So, one pays an effective annual rate of


0.24 12
1 = 0.2682.
1+
12

Albert Cohen (MSU)

MATH 360: Theory of Investment and Credit

(20)

MSU Spring 2014

27 / 223

Example 1.9

Which is better:
(A) 15.25% compounded semi-annually, or
(B) 15% compounded monthly ?

Albert Cohen (MSU)

MATH 360: Theory of Investment and Credit

MSU Spring 2014

28 / 223

Example 1.9

Which is better:
(A) 15.25% compounded semi-annually, or
(B) 15% compounded monthly ?

0.1525 2
1 = 0.1583
2


0.15 12
iB (eff ) = 1 +
1 = 0.1608
12


iA (eff ) =

Albert Cohen (MSU)

1+

MATH 360: Theory of Investment and Credit

MSU Spring 2014

(21)

28 / 223

Notation

Actuaries reserve i for effective annual rate and i (m) for the nominal rate
compounded m times annually. The link between the two is
!m
i (m)
i = 1+
1
m

Albert Cohen (MSU)

MATH 360: Theory of Investment and Credit

MSU Spring 2014

29 / 223

Notation

Actuaries reserve i for effective annual rate and i (m) for the nominal rate
compounded m times annually. The link between the two is
!m
i (m)
i = 1+
1
m


1
i (m) = m (1 + i) m 1

Albert Cohen (MSU)

MATH 360: Theory of Investment and Credit

MSU Spring 2014

29 / 223

Notation

Actuaries reserve i for effective annual rate and i (m) for the nominal rate
compounded m times annually. The link between the two is
!m
i (m)
i = 1+
1
m


1
i (m) = m (1 + i) m 1
i () = lim i (m) = ln (1 + i)
m

Albert Cohen (MSU)

MATH 360: Theory of Investment and Credit

MSU Spring 2014

29 / 223

Notation

Actuaries reserve i for effective annual rate and i (m) for the nominal rate
compounded m times annually. The link between the two is
!m
i (m)
i = 1+
1
m


1
i (m) = m (1 + i) m 1
(22)
i () = lim i (m) = ln (1 + i)
m
(m)

<i

Albert Cohen (MSU)

MATH 360: Theory of Investment and Credit

MSU Spring 2014

29 / 223

Example 1.10

Assume i = 0.12. Then


Table: Example 1.10

m
1
2
6
52
365

Albert Cohen (MSU)



1
m (1 + i) m 1
0.12
0.1166
0.1144
0.1135
0.113346
0.113329

MATH 360: Theory of Investment and Credit

MSU Spring 2014

30 / 223

Comparison of continuous to compound interest earned

Assume that we are promised 12% annual interest, and we wish to


calculate the daily interest gained on 10, 000, 000:
1
= 3287.67
interestovernight = 10, 000, 000 0.12
 1 365
interestcontinuous = 10, 000, 000 e 365 1 = 3288.21,

(23)

a difference of 0.54 on a principle of 10 million

Albert Cohen (MSU)

MATH 360: Theory of Investment and Credit

MSU Spring 2014

31 / 223

Discount Rates
Sometimes, interest paid up front: Receive A(0) up front, pay back
A(1) > A(0).
Consider the example
A(0) = 900
A(1) = 1000
A(1) A(0)
d=
A(1)

Albert Cohen (MSU)

MATH 360: Theory of Investment and Credit

MSU Spring 2014

32 / 223

Discount Rates
Sometimes, interest paid up front: Receive A(0) up front, pay back
A(1) > A(0).
Consider the example
A(0) = 900
A(1) = 1000
A(1) A(0)
d=
A(1)
1
= =1d
1+i

(24)

Essentially, the discount is the forward price of a dollar (or any other unit
of currency) in the interest market with no carrying cost or dividends
(coupons) paid out.

Albert Cohen (MSU)

MATH 360: Theory of Investment and Credit

MSU Spring 2014

32 / 223

Example 1.11
A T-bill represents the forward value today of 100 delivered at time T .
The price is calculated via simple discount:


T
P(0) = P(T ) (1 d t) = 100 1 d
365

Albert Cohen (MSU)

MATH 360: Theory of Investment and Credit

MSU Spring 2014

33 / 223

Example 1.11
A T-bill represents the forward value today of 100 delivered at time T .
The price is calculated via simple discount:


T
P(0) = P(T ) (1 d t) = 100 1 d
365


(25)
T
100 = P(T ) = P(0) (1 + i t) = P(0) 1 + i
365

Albert Cohen (MSU)

MATH 360: Theory of Investment and Credit

MSU Spring 2014

33 / 223

Example 1.11
A T-bill represents the forward value today of 100 delivered at time T .
The price is calculated via simple discount:


T
P(0) = P(T ) (1 d t) = 100 1 d
365


(25)
T
100 = P(T ) = P(0) (1 + i t) = P(0) 1 + i
365
Table: Example 1.11

Term
12day
28day
91day
128day
Albert Cohen (MSU)

Discount (%)
0.965
0.9940
1.130
1.400

Investment (%)
0.974
0.952
1.150
1.430

MATH 360: Theory of Investment and Credit

Price per 100


99.680
99.927
99.174
99.292
MSU Spring 2014

33 / 223

Nominal Annual Discount Rate

d (m) is the quoted annual discount rate that is applied m times over the
(m)
year, with the effective discount rate as dm for the period of m1 years.
Also, d is the effective discount rate :
!m
d (m)
1d = 1
m


1
(26)
d (m) = m 1 (1 d) m


1
d () = lim d (m) = ln
= i ()
m
1d

Albert Cohen (MSU)

MATH 360: Theory of Investment and Credit

MSU Spring 2014

34 / 223

Force of Interest
From time t1 to t2 , an investment grows by a rate of
it1 t2 :=

A(t2 ) A(t1 )
A(t1 )

(27)

Now, fix a time t1 = t and correspondingly, let t2 = t + m1 . It follows that


the nominal rate that gives the same annual growth from t to t + m1 is

i (m) = m

Albert Cohen (MSU)

A t+

1
m

A t+
A(t)
1
=

A(t)
A(t)

MATH 360: Theory of Investment and Credit

1
m
1
m

A(t)

MSU Spring 2014

35 / 223

Force of Interest
From time t1 to t2 , an investment grows by a rate of
it1 t2 :=

A(t2 ) A(t1 )
A(t1 )

(27)

Now, fix a time t1 = t and correspondingly, let t2 = t + m1 . It follows that


the nominal rate that gives the same annual growth from t to t + m1 is

i (m) = m
lim i (m) =

Albert Cohen (MSU)

1
A(t)

1
m

A t+
A(t)
1
=

A(t)
A(t)

1
A t + m A(t)
lim
1

A t+

1
m
1
m

A(t)

MATH 360: Theory of Investment and Credit

MSU Spring 2014

35 / 223

Force of Interest
From time t1 to t2 , an investment grows by a rate of
it1 t2 :=

A(t2 ) A(t1 )
A(t1 )

(27)

Now, fix a time t1 = t and correspondingly, let t2 = t + m1 . It follows that


the nominal rate that gives the same annual growth from t to t + m1 is

i (m) = m

1
A(t) m
1 dA
=
A(t) dt

lim i (m) =

1
m

Albert Cohen (MSU)

A t+
A(t)
1
=

A(t)
A(t)

1
A t + m A(t)
lim
1

A t+

1
m
1
m

A(t)

MATH 360: Theory of Investment and Credit

MSU Spring 2014

35 / 223

Force of Interest
From time t1 to t2 , an investment grows by a rate of
it1 t2 :=

A(t2 ) A(t1 )
A(t1 )

(27)

Now, fix a time t1 = t and correspondingly, let t2 = t + m1 . It follows that


the nominal rate that gives the same annual growth from t to t + m1 is

i (m) = m

1
A(t) m
1 dA
=
=: (t)
A(t) dt

lim i (m) =

1
m

A t+
A(t)
1
=

A(t)
A(t)

1
A t + m A(t)
lim
1

A t+

Albert Cohen (MSU)

1
m
1
m

A(t)

(28)

MATH 360: Theory of Investment and Credit

MSU Spring 2014

35 / 223

Force of Interest

Since (t) =
Z t
0

1 dA
A(t) dt

d
= dt
[ln (A(t))], it follows that
Z t
d
[ln (A(s))] ds = ln (A(t)) ln (A(0))
(s)ds =
0 ds

(29)

and so
A(t) = A(0)e

Albert Cohen (MSU)

Rt
0

(s)ds

MATH 360: Theory of Investment and Credit

(30)

MSU Spring 2014

36 / 223

Some Examples...

Calculate (t) for


A(t) = e t

A(t) = t e t
A(t) = (t 1)2
A(t) = A(0) (1 + i t)
A(t) = A(0) (1 + i)t

Albert Cohen (MSU)

MATH 360: Theory of Investment and Credit

MSU Spring 2014

37 / 223

Example 1.14

Assume (t) = 0.08 + 0.005t and your assets initial value is A(0) = 1000.
Then
A(t) = A(0)e

Rt

0 (0.08+0.005s)ds

= 1000e 0.08t+0.0025t

A(5) = 1000e 0.085+0.00255 = 1588.04

(31)

But, if we were give A(2) = 1000, then


A(7) = A(2)e

Albert Cohen (MSU)

R7
2

(s)ds

2 22 )

= 1000e 0.085+0.0025(7

MATH 360: Theory of Investment and Credit

= 1669.46

MSU Spring 2014

(32)

38 / 223

Some thoughts...

This limiting term, known as the force of interest represents the


theoretical upper limit for interest gained on this asset. However, In the
derivation above, there is no randomness involved in obtaining the ordinary
differential equation
dA(t) = (t)A(t)dt

(33)

To handle this lack of stochasticity, one common asset model (Geometric


Brownian Motion) is defined by the stochastic differential equation
dA(t) = (t)A(t)dt + (A(t)) dW (t),

(34)

where W (.) is the Wiener process.

Albert Cohen (MSU)

MATH 360: Theory of Investment and Credit

MSU Spring 2014

39 / 223

Introduction and Geometric Series

An annuity is a series of periodic payments.

Albert Cohen (MSU)

MATH 360: Theory of Investment and Credit

MSU Spring 2014

40 / 223

Introduction and Geometric Series

An annuity is a series of periodic payments. In this class, they are not


event-contingent, but rather only depend on the passage of time.

Albert Cohen (MSU)

MATH 360: Theory of Investment and Credit

MSU Spring 2014

40 / 223

Introduction and Geometric Series

An annuity is a series of periodic payments. In this class, they are not


event-contingent, but rather only depend on the passage of time. To
compute the value of the most foundational annuity, we must recall the
following fact:

Albert Cohen (MSU)

MATH 360: Theory of Investment and Credit

MSU Spring 2014

40 / 223

Introduction and Geometric Series

An annuity is a series of periodic payments. In this class, they are not


event-contingent, but rather only depend on the passage of time. To
compute the value of the most foundational annuity, we must recall the
following fact: If x R : x 6= 1, then
Xn := 1 + x + ... + x n

Albert Cohen (MSU)

MATH 360: Theory of Investment and Credit

MSU Spring 2014

40 / 223

Introduction and Geometric Series

An annuity is a series of periodic payments. In this class, they are not


event-contingent, but rather only depend on the passage of time. To
compute the value of the most foundational annuity, we must recall the
following fact: If x R : x 6= 1, then
Xn := 1 + x + ... + x n
(1 x) Xn = 1 x n+1

Albert Cohen (MSU)

MATH 360: Theory of Investment and Credit

MSU Spring 2014

40 / 223

Introduction and Geometric Series

An annuity is a series of periodic payments. In this class, they are not


event-contingent, but rather only depend on the passage of time. To
compute the value of the most foundational annuity, we must recall the
following fact: If x R : x 6= 1, then
Xn := 1 + x + ... + x n
(1 x) Xn = 1 x n+1

(35)

1 x n+1
Xn =
1x

Albert Cohen (MSU)

MATH 360: Theory of Investment and Credit

MSU Spring 2014

40 / 223

Example 2.1

The federal government sends Smith a family allowance of 30 every month


for Smiths child. Smith deposits the payments in bank account on the
last day of each month. The account earns interest at the annual rate of
9% compounded monthly and payable on the last day of each month, on
the minimum monthly balance. If the first payment is deposited on May
31, 1998, what is the account balance on December 31,2009, including the
payment just made?

Albert Cohen (MSU)

MATH 360: Theory of Investment and Credit

MSU Spring 2014

41 / 223

Example 2.1

There are 140 total payments

Albert Cohen (MSU)

MATH 360: Theory of Investment and Credit

MSU Spring 2014

42 / 223

Example 2.1

There are 140 total payments


imonthly =

i
12

Albert Cohen (MSU)

= 0.0075

MATH 360: Theory of Investment and Credit

MSU Spring 2014

42 / 223

Example 2.1

There are 140 total payments


imonthly =

i
12

= 0.0075

TotalValue = 30 + 30 (1.0075)1 + ... + 30 (1.0075)139


= 30

139
X
k=0

1.0075k = 30

1 1.0075140
1 1.0075

(36)

= 7385.91

Albert Cohen (MSU)

MATH 360: Theory of Investment and Credit

MSU Spring 2014

42 / 223

Level Payment Annuities

Number of payments in a series of payments is called the term of the


annuity
Time between the successive payments is called the payment period,
or frequency
A series of payments whose value is found at the time of the final
payment is known as accumulated annuity immediate

Albert Cohen (MSU)

MATH 360: Theory of Investment and Credit

MSU Spring 2014

43 / 223

Level Payment Annuities

Number of payments in a series of payments is called the term of the


annuity
Time between the successive payments is called the payment period,
or frequency
A series of payments whose value is found at the time of the final
payment is known as accumulated annuity immediate

sn i :=

n1
X
k=0

Albert Cohen (MSU)

(1 + i)k =

(1 + i)n 1
i

MATH 360: Theory of Investment and Credit

(37)

MSU Spring 2014

43 / 223

Level Payment Annuities

Number of payments in a series of payments is called the term of the


annuity
Time between the successive payments is called the payment period,
or frequency
A series of payments whose value is found at the time of the final
payment is known as accumulated annuity immediate

sn i :=

n1
X

(1 + i)k =

k=0

(1 + i)n 1
i

(37)

Equivalently, (1 + i)n = 1 + i sn i .

Albert Cohen (MSU)

MATH 360: Theory of Investment and Credit

MSU Spring 2014

43 / 223

Ex 2.2

What level amount must be deposited on May 1 and Nov 1 each year from
1998 to 2005, inclusive, to accumulate to 7000 on November 1, 2005 if
the nominal annual interest rate, compounded semi-annually, is 9% ?

Albert Cohen (MSU)

MATH 360: Theory of Investment and Credit

MSU Spring 2014

44 / 223

Interest Compounded Monthly

16 total deposits
i (2) = 0.09
X denotes the level amount deposited per period

Albert Cohen (MSU)

MATH 360: Theory of Investment and Credit

MSU Spring 2014

45 / 223

Interest Compounded Monthly

16 total deposits
i (2) = 0.09
X denotes the level amount deposited per period
7000 = X 1 + (1.045)1 + ... + (1.045)15

= X s16 0.045

(38)

X = 308.11

Albert Cohen (MSU)

MATH 360: Theory of Investment and Credit

MSU Spring 2014

45 / 223

Example 2.3

Suppose that in Example 2.1, Smiths child is born in April 1998 and the
first payment is received in May (and deposited at the end of May.) The
payments continue and the deposits are made at the end of the month
until, and including the month of, the childs 16th birthday. The payments
stop after the 16th birthday, but the balance continues to accumulate with
interest until the end of the month of the childs 21st birthday. What is
the balance X in the account at that time ?

Albert Cohen (MSU)

MATH 360: Theory of Investment and Credit

MSU Spring 2014

46 / 223

Example 2.3

Suppose that in Example 2.1, Smiths child is born in April 1998 and the
first payment is received in May (and deposited at the end of May.) The
payments continue and the deposits are made at the end of the month
until, and including the month of, the childs 16th birthday. The payments
stop after the 16th birthday, but the balance continues to accumulate with
interest until the end of the month of the childs 21st birthday. What is
the balance X in the account at that time ?
X = 1.007560 30 s192 0.0075 = 20, 028.68

Albert Cohen (MSU)

MATH 360: Theory of Investment and Credit

MSU Spring 2014

(39)

46 / 223

Some Arithmetic

After accumulating cash via a series of payments at rate i until time n, we


allow this amount to grow until time n + k :
(1 + i)n 1
(1 + i)k
i
(1 + i)k

Value(n + k) = sn i (1 + i)k =
=

(1 + i)n+k

i
(1 + i)n+k 1 (1 + i)k 1
=

i
i
= sn+k i sk i

Albert Cohen (MSU)

MATH 360: Theory of Investment and Credit

MSU Spring 2014

47 / 223

Some Arithmetic

After accumulating cash via a series of payments at rate i until time n, we


allow this amount to grow until time n + k :
(1 + i)n 1
(1 + i)k
i
(1 + i)k

Value(n + k) = sn i (1 + i)k =
=

(1 + i)n+k

i
(1 + i)n+k 1 (1 + i)k 1
=

i
i
= sn+k i sk i

(40)

sn+k i = sk i + sn i (1 + i)k

Albert Cohen (MSU)

MATH 360: Theory of Investment and Credit

MSU Spring 2014

47 / 223

Example 2.4

Suppose that in Example 2.1, the nominal annual interest rate earned on
the account changes to 7.5%, still compounded monthly, as of January
2004. What is the accumulated value of the account on December 31
2009 ?

Albert Cohen (MSU)

MATH 360: Theory of Investment and Credit

MSU Spring 2014

48 / 223

Example 2.4

Suppose that in Example 2.1, the nominal annual interest rate earned on
the account changes to 7.5%, still compounded monthly, as of January
2004. What is the accumulated value of the account on December 31
2009 ?

Value = 30 s68 0.0075 1.0062572 + s72 0.00625 = 6865.22

Albert Cohen (MSU)

MATH 360: Theory of Investment and Credit

MSU Spring 2014

(41)

48 / 223

Example 2.5

Suppose 10 monthly payments of 50 each are followed by 14 monthly


payments of 75 each. If interest is at an effective monthly rate of 1%, what
is the accumulated value of the series at the time of the final payment ?

Albert Cohen (MSU)

MATH 360: Theory of Investment and Credit

MSU Spring 2014

49 / 223

Example 2.5

Suppose 10 monthly payments of 50 each are followed by 14 monthly


payments of 75 each. If interest is at an effective monthly rate of 1%, what
is the accumulated value of the series at the time of the final payment ?
Value = 50 s24 0.01 + 25 s14 0.01

Albert Cohen (MSU)

MATH 360: Theory of Investment and Credit

(42)

MSU Spring 2014

49 / 223

Present Value of an Annuity


Make a lump sum payment X now to receive periodic payments C ,
starting one period from today. If the market bears a constant interest of
i, the the the Present Value of this Annuity Immediate is calculated as
C
C
C
+
+ ... +
2
1+i
(1 + i)
(1 + i)n
n
n1
X
X
=C
k = C
k

X =

k=1

k=0

(43)

1 n
=C
1
1 n
1 (1 + i)n
=C
=C
i
i
C an i

Albert Cohen (MSU)

MATH 360: Theory of Investment and Credit

MSU Spring 2014

50 / 223

Example 2.7

Brown has bought a new car and requires a loan of 12000 to pay for it.
The car dealer offers Brown two alternatives on the loan:
A : Monthly payments for 3 years, starting one month after purchase,
with an annual interest rate of 12% compounded monthly
B : Monthly payments for 4 years, also starting one month after
purchase, with an annual interest rate of 15% compounded monthly.
Find Browns monthly payment and the total amount paid over the course
of the repayment period under each of the two options.

Albert Cohen (MSU)

MATH 360: Theory of Investment and Credit

MSU Spring 2014

51 / 223

Example 2.7

12000 = PA a36 0.01 PA = 398.57


12000 = PB a48 0.0125 PB = 333.97
TotalValue(A) = 36 PA = 14348.52

(44)

TotalValue(B) = 48 PB = 16030.56

Albert Cohen (MSU)

MATH 360: Theory of Investment and Credit

MSU Spring 2014

52 / 223

Example 2.8

Suppose that in Example 2.7, Brown can repay the loan, still with 36
payments under option A or 48 payments under option B, with the first
payment made 9 months after the car is purchased in either case.
Assuming interest accrues from the time of the car purchase, find the
payments required under options A and B. This is known as a deferred
annuity

Albert Cohen (MSU)

MATH 360: Theory of Investment and Credit

MSU Spring 2014

53 / 223

Example 2.8

12000 = PA0 1.019 + 1.0110 + ... + 1.0144

= PA0 A8 a36 0.01


PA0 = 431.60
12000 = PB0 1.01259 + 1.012510 + ... + 1.012556

(45)

= PB0 B8 a48 0.0125


PB0 = 368.86

Albert Cohen (MSU)

MATH 360: Theory of Investment and Credit

MSU Spring 2014

54 / 223

Some Facts About Annuities

There is a duality between an i and sn i :


sn i = (1 + i)n an i

(46)

an i = n s n i
As n , we have
a i = lim an i
n

(47)

1 n
1
=
n
i
i

= lim

Albert Cohen (MSU)

MATH 360: Theory of Investment and Credit

MSU Spring 2014

55 / 223

Example 2.9

A perpetuity immediate pays X per year. Brian receives the first n


payments, Colleen receives the next n payments, and Jeff receives the
remaining payments. Brians share of the present value of the original
perpetuity is 40%, and Jeffs share is K . Calculate K .

Albert Cohen (MSU)

MATH 360: Theory of Investment and Credit

MSU Spring 2014

56 / 223

Example 2.9

1 n
i
X
= 0.4 X a i = 0.4
i
1 n = 0.4

PV (Brian) = X an i = X

PV (Colleen) = n X an i = 0.4 0.6

Albert Cohen (MSU)

X
X
= 0.24
i
i

MATH 360: Theory of Investment and Credit

MSU Spring 2014

57 / 223

Example 2.9

1 n
i
X
= 0.4 X a i = 0.4
i
1 n = 0.4

PV (Brian) = X an i = X

PV (Colleen) = n X an i = 0.4 0.6

X
X
= 0.24
i
i

(48)

X
= PV = PV (Brian) + PV (Jeff ) + PV (Colleen)
i
X
X
= 0.4 + K + 0.24
i
i
X
K = 0.36
i
Albert Cohen (MSU)

MATH 360: Theory of Investment and Credit

MSU Spring 2014

57 / 223

Recall...

sn i = 1 + (1 + i) + (1 + i)2 + ... + (1 + i)n1 =


an i

1 n
= + = ... + =
i
2

(1 + i)n 1
i

(49)

Of course, sn i is the accumulation of all the payments at the final time,


whereas an i is the present value of all the paymenst, one period before the
first payment. Now, define
sn i := (1 + i) + (1 + i)2 + ... + (1 + i)n = (1 + i)sn i
= annuity due
= accummulated value one-period after final payment

(50)

an i := (1 + i)an i
= present value at time of first payment
Albert Cohen (MSU)

MATH 360: Theory of Investment and Credit

MSU Spring 2014

58 / 223

Level Payment Annuities: Some Generalizations.

What happens when compounding interest period and annuity payment


period dont coincide? For example: Consider 4 deposits made per year,
over 16 years, of 1000 each. What is the balance after the last payment if
interest is quoted at 9% nominal annual interest rate, compounded
monthly?

Albert Cohen (MSU)

MATH 360: Theory of Investment and Credit

MSU Spring 2014

59 / 223

Level Payment Annuities: Some Generalizations.

What happens when compounding interest period and annuity payment


period dont coincide? For example: Consider 4 deposits made per year,
over 16 years, of 1000 each. What is the balance after the last payment if
interest is quoted at 9% nominal annual interest rate, compounded
monthly?
In this case, the equivalent rate j for the 3month periods satisfies
3
1 + j = 1 + 0.09
. It follows that
12
Value = 1000s64 j = 141, 076

Albert Cohen (MSU)

MATH 360: Theory of Investment and Credit

(51)

MSU Spring 2014

59 / 223

mthly payable annuities: Some Generalizations.


Imagine that for each of n periods in the term of an annuity, 1 is paid out
over m payments of m1 each. Then the present value is computed using
(m)
j = i m , where
(m)

1
1
1
sm j + 2 sm j + ... + n sm j
m
m
m


m
i (m)
1+ m
1

1
= + 2 + ... + n
i (m)
m
m

m
(m)
1 + im
1
i
= an i
= an i (m)
(m)
i
i

an i =

Albert Cohen (MSU)

MATH 360: Theory of Investment and Credit

MSU Spring 2014

60 / 223

mthly payable annuities: Some Generalizations.


Imagine that for each of n periods in the term of an annuity, 1 is paid out
over m payments of m1 each. Then the present value is computed using
(m)
j = i m , where
(m)

1
1
1
sm j + 2 sm j + ... + n sm j
m
m
m


m
i (m)
1+ m
1

1
= + 2 + ... + n
i (m)
m
m

m
(m)
1 + im
1
i
= an i
= an i (m)
(m)
i
i
i
= sn i (m)
i

an i =

(m)

sn i

Albert Cohen (MSU)

MATH 360: Theory of Investment and Credit

MSU Spring 2014

(52)

60 / 223

mthly payable annuities: Some Generalizations.

As m , we have i (m) ln (1 + i), and so it follows that


(m)

i
ln (1 + i)
i

ln (1 + i)

an i an i
(m)

sn i sn i

(53)

Lets analyze this using calculus!

Albert Cohen (MSU)

MATH 360: Theory of Investment and Credit

MSU Spring 2014

61 / 223

mthly payable annuities: Some Generalizations.

Take t2 t1 = m1 , and assume that 1 is paid out at a continuous rate.


Then, from t to t + dt, the amount paid out is dt. At time n, the value of
dt accumulated from t to n is dt (1 + i)nt .

Albert Cohen (MSU)

MATH 360: Theory of Investment and Credit

MSU Spring 2014

62 / 223

mthly payable annuities: Some Generalizations.

Take t2 t1 = m1 , and assume that 1 is paid out at a continuous rate.


Then, from t to t + dt, the amount paid out is dt. At time n, the value of
dt accumulated from t to n is dt (1 + i)nt . Add up all of these bits and
we obtain
Z n
Z n
sn i =
(1 + i)nt dt =
e ln (1+i)(nt) dt
0
0
#n
"

ln
(1+i)t
e
= e ln (1+i)n
(54)
ln (1 + i)
0

(1 + i)n 1
i
(m)
=
= sn i
= lim s
ln (1 + i)
ln (1 + i) m n i

Albert Cohen (MSU)

MATH 360: Theory of Investment and Credit

MSU Spring 2014

62 / 223

An Example: Daily vs Continuous deposits

Consider depositing 12 per day in 2004 and 2005 and 15 per day in 2006.
The earned interest in 2004-05 is i1 = 9% effective per year, and i2 = 12%
effective per year in 2006. Compute the total accumulated amount at the
end of 2006 if computed via (a) daily deposits and (b) continuous
deposits. Assume no leap years!

Albert Cohen (MSU)

MATH 360: Theory of Investment and Credit

MSU Spring 2014

63 / 223

An Example: Daily vs Continuous deposits

For daily deposits, we compute the equivalent daily interest rates j1 for
2004-05 and j2 for 2006 by
1

j1 = (1.09) 365 1 = 0.00023631

(55)

j2 = (1.12) 365 1 = 0.00031054


So,
AccValue = 12 s730 j1 (1 + i2 ) + 15 s365 j2
= 12 s730 0.00023631 (1.12) + 15 s365 0.00031054

(56)

= 16502.59

Albert Cohen (MSU)

MATH 360: Theory of Investment and Credit

MSU Spring 2014

64 / 223

An Example: Daily vs Continuous deposits

For continuous deposits, we compute that 12 365 = 4380 is invested per


yearfor the period 2004-05 and 15 365 = 5475 for 2006.
So,
AccValue = 4380 s2 0.09 (1.12) + 5475 s1 0.12
1.092 1
1.12 1
(1.12) + 5475
ln (1.09)
ln (1.12)
= 16504.75
= 4380

Albert Cohen (MSU)

MATH 360: Theory of Investment and Credit

MSU Spring 2014

(57)

65 / 223

Continuous Annuities in General


Define a(t1 , t2 ) as the accumulated value at t2 of an amount 1 invested at
t1 .

Albert Cohen (MSU)

MATH 360: Theory of Investment and Credit

MSU Spring 2014

66 / 223

Continuous Annuities in General


Define a(t1 , t2 ) as the accumulated value at t2 of an amount 1 invested at
t1 . Hence,
Z t2
a(t, t2 )dt
(58)
t1

is the accumulated value at t2 of a continuous annuity paying 1 per


unit time over the interval (t1 , t2 ),
and
Z t2
1
dt
(59)
t1 a(t, t2 )
is the present value at t1 of a continuous annuity paying 1 per unit time
over the interval (t1 , t2 ),

Albert Cohen (MSU)

MATH 360: Theory of Investment and Credit

MSU Spring 2014

66 / 223

Continuous Annuities in General

If s is the force of interest at time s, then


Rt

ds

a(t1 , t2 ) = e t1 s
Z n R
t
e 0 s ds dt
an s =
Z0 n R
n
sn s =
e t s ds dt

(60)

Albert Cohen (MSU)

MATH 360: Theory of Investment and Credit

MSU Spring 2014

67 / 223

Unknown Number of Payments in an Annuity

Assuming n level payments of J each at interest rate i, the accumulated


value is

Albert Cohen (MSU)

MATH 360: Theory of Investment and Credit

MSU Spring 2014

68 / 223

Unknown Number of Payments in an Annuity

Assuming n level payments of J each at interest rate i, the accumulated


value is


M = J (1 + i)n1 + ... + (1 + i) + 1
(1 + i)n 1
=J
= J sn i
i

Albert Cohen (MSU)

MATH 360: Theory of Investment and Credit

MSU Spring 2014

68 / 223

Unknown Number of Payments in an Annuity

Assuming n level payments of J each at interest rate i, the accumulated


value is


M = J (1 + i)n1 + ... + (1 + i) + 1
(1 + i)n 1
=J
= J sn i
i
(61)
M
(1 + i)n = 1 + i
J

ln 1 + i M
J
n=
ln 1 + i

Albert Cohen (MSU)

MATH 360: Theory of Investment and Credit

MSU Spring 2014

68 / 223

Example 2.13

Smith wishes to accumulate 1000 by means of semi-annual deposits


earning interest at nominal rate i (2) = 0.08, with interest creditied
semi-annually. Regular deposits of 50 are made. Find the number of
periods and regular deposits needed, along with any additional fractional
deposits if such a fractional deposit is made at the time of the last regular
deposit or the very next period (i.e. last regular deposit can be 0.)

Albert Cohen (MSU)

MATH 360: Theory of Investment and Credit

MSU Spring 2014

69 / 223

Example 2.13

Here, the parameters are M = 1000, J = 50, i = 0.04. It follows that



ln 1 + i M
J
= 14.9866
(62)
n=
ln 1 + i

Albert Cohen (MSU)

MATH 360: Theory of Investment and Credit

MSU Spring 2014

70 / 223

Example 2.13

Here, the parameters are M = 1000, J = 50, i = 0.04. It follows that



ln 1 + i M
J
= 14.9866
(62)
n=
ln 1 + i
Rounding down to n = 14, we obtain 50s14 0.04 = 914.60..

Albert Cohen (MSU)

MATH 360: Theory of Investment and Credit

MSU Spring 2014

70 / 223

Example 2.13

Here, the parameters are M = 1000, J = 50, i = 0.04. It follows that



ln 1 + i M
J
= 14.9866
(62)
n=
ln 1 + i
Rounding down to n = 14, we obtain 50s14 0.04 = 914.60.. However, 50 is
not enough to carry over to get a final amount of 1000, so let the 914.60
accrue for 1 period to 914.60 (1.04) = 951.18 , and then add the
fractional payment of 48.82. Hence 15 periods are needed

Albert Cohen (MSU)

MATH 360: Theory of Investment and Credit

MSU Spring 2014

70 / 223

Example 2.13

Smith pays 100 per month to a fund earning i (12) = 0.09, with interest
credited on the last day of each month. At the time of each deposit, 10 is
deducted from the deposit for expenses and administration fees. The first
deposit is made on the last day of Jan 2000. In which month does the
accumulated value become greater than the total gross contribution to
that point?

Albert Cohen (MSU)

MATH 360: Theory of Investment and Credit

MSU Spring 2014

71 / 223

Example 2.13
We wish to solve for the time when the Value of the account exceeds the
total amount invested, i.e. the value n that satisfies


90 1 + 1.0075 + ... + 1.0075n1 100n

Albert Cohen (MSU)

MATH 360: Theory of Investment and Credit

MSU Spring 2014

72 / 223

Example 2.13
We wish to solve for the time when the Value of the account exceeds the
total amount invested, i.e. the value n that satisfies


90 1 + 1.0075 + ... + 1.0075n1 100n
100
n
sn i
90

Albert Cohen (MSU)

MATH 360: Theory of Investment and Credit

MSU Spring 2014

72 / 223

Example 2.13
We wish to solve for the time when the Value of the account exceeds the
total amount invested, i.e. the value n that satisfies


90 1 + 1.0075 + ... + 1.0075n1 100n
100
(63)
n
sn i
90
1.0075n 1 + 0.00833n

Albert Cohen (MSU)

MATH 360: Theory of Investment and Credit

MSU Spring 2014

72 / 223

Example 2.13
We wish to solve for the time when the Value of the account exceeds the
total amount invested, i.e. the value n that satisfies


90 1 + 1.0075 + ... + 1.0075n1 100n
100
(63)
n
sn i
90
1.0075n 1 + 0.00833n
Essentially, we are solving a fixed point equation for f (x) = 0, where
f (x) = e kx 1 0.00833x

Albert Cohen (MSU)

MATH 360: Theory of Investment and Credit

MSU Spring 2014

72 / 223

Example 2.13
We wish to solve for the time when the Value of the account exceeds the
total amount invested, i.e. the value n that satisfies


90 1 + 1.0075 + ... + 1.0075n1 100n
100
(63)
n
sn i
90
1.0075n 1 + 0.00833n
Essentially, we are solving a fixed point equation for f (x) = 0, where
f (x) = e kx 1 0.00833x

(64)

k = ln (1.0075)

Albert Cohen (MSU)

MATH 360: Theory of Investment and Credit

MSU Spring 2014

72 / 223

Example 2.13
We wish to solve for the time when the Value of the account exceeds the
total amount invested, i.e. the value n that satisfies


90 1 + 1.0075 + ... + 1.0075n1 100n
100
(63)
n
sn i
90
1.0075n 1 + 0.00833n
Essentially, we are solving a fixed point equation for f (x) = 0, where
f (x) = e kx 1 0.00833x

(64)

k = ln (1.0075)
In this case, plotting the above function of x gives an intercept of
approximately 28.6. Hence, n = 29.

Albert Cohen (MSU)

MATH 360: Theory of Investment and Credit

MSU Spring 2014

72 / 223

Dividend Discount Model for Stock Shares

We now have a philosophy for pricing assets that bring a future stream of
payments. Simply put, the value today of the asset is simply the Net
Present Value, opr NPV, of that stream of payments, when adjusted for
inflation or (compound) growth, if applicable.

Albert Cohen (MSU)

MATH 360: Theory of Investment and Credit

MSU Spring 2014

73 / 223

Dividend Discount Model for Stock Shares


If a table, or number line of future payments looks like
Table: Example 1.5: Payments In

Week
Payment in

0
0

1
K

2
K (1 + r )

...n
K (1 + r )n1

then the NPV, valued with rate i, is


K
K (1 + r )
K
+
+ ... +
1+i
(1 + i)2

n1 
X
K
K
1+r j

=
=
1+i
1+i
1+i

NPV =

j=0

Albert Cohen (MSU)

(1 + r )n1
(1 + i)n

n
1 1+r
1+i

1 1+r
1+i

MATH 360: Theory of Investment and Credit

MSU Spring 2014

(65)

74 / 223

Dividend Discount Model for Stock Shares

As n , we approach a perpetuity, and


NPV

K
1
K

=
1+r
1 + i 1 1+i
i r

(66)

In words, we have a perpetuity that is adjusted for the growth rate r . We


use this formula to price a stock or any other asset as a perpetuity with
possible compound growth at rate r .

Albert Cohen (MSU)

MATH 360: Theory of Investment and Credit

MSU Spring 2014

75 / 223

Example 2.13
Stock X pays dividend 50 per year with growth of 5% after the first year.
John purchases X at the theoretical price corresponding to an effective
yield of 10%. After receiving the the 10th dividend, John sells the stock for
price P. If annual yield for John was 8%, what is the fair price for P ?

Albert Cohen (MSU)

MATH 360: Theory of Investment and Credit

MSU Spring 2014

76 / 223

Example 2.13
Stock X pays dividend 50 per year with growth of 5% after the first year.
John purchases X at the theoretical price corresponding to an effective
yield of 10%. After receiving the the 10th dividend, John sells the stock for
price P. If annual yield for John was 8%, what is the fair price for P ?
The equation of value is
9

50 X
50
= 1000 =

0.10 0.05
1.08
j=0

Albert Cohen (MSU)

1.05
1.08

j

MATH 360: Theory of Investment and Credit

P
1.0810

MSU Spring 2014

76 / 223

Example 2.13
Stock X pays dividend 50 per year with growth of 5% after the first year.
John purchases X at the theoretical price corresponding to an effective
yield of 10%. After receiving the the 10th dividend, John sells the stock for
price P. If annual yield for John was 8%, what is the fair price for P ?
The equation of value is
9

50 X
50
= 1000 =

0.10 0.05
1.08
j=0

P = 1000(1.08)

10

1.05
1.08

j

50(1.08)

P
1.0810


9 
X
1.05 j
j=0

= 1000(1.08)10 50(1.08)9

1
1

1.08
 !
1.05 10

(67)

1.08

1.05 1
1.08

= 1275.54.
Albert Cohen (MSU)

MATH 360: Theory of Investment and Credit

MSU Spring 2014

76 / 223

Increasing Annuities
Consider now a payment schedule where the payment, at time k, was
Km = m .

Albert Cohen (MSU)

MATH 360: Theory of Investment and Credit

MSU Spring 2014

77 / 223

Increasing Annuities
Consider now a payment schedule where the payment, at time k, was
Km = m .
Now, the net present value is
X =

n
X

Km

m=1

n
X
m=1

n
X
m=1

m
(1 + i)m

(68)

Then

Albert Cohen (MSU)

MATH 360: Theory of Investment and Credit

MSU Spring 2014

77 / 223

Increasing Annuities
Consider now a payment schedule where the payment, at time k, was
Km = m .
Now, the net present value is
X =

n
X

Km

m=1

n
X

m=1

n
X
m=1

m
(1 + i)m

(68)

Then
(1 + i) X =

n
X

m m1 =

m=1

n
X
m=1

m
(1 + i)m1

i X = 1 + + + ... + n1 n n
1 n
=
n n
1
an i n n
X =
=: (Ia)n i
i
Albert Cohen (MSU)

MATH 360: Theory of Investment and Credit

MSU Spring 2014

(69)

77 / 223

Increasing Annuities

An increasing perpetuity is defined as


lim (Ia)n i = lim

1 n
1

n n
i

(70)
1+i
= 2
i
Notice that the level payment perpetuity of Kl 1 has present value of
only 1i . Since i 0, we have i12 >> 1i .

Albert Cohen (MSU)

MATH 360: Theory of Investment and Credit

MSU Spring 2014

78 / 223

Increasing Annuities

An increasing perpetuity is defined as


lim (Ia)n i = lim

1 n
1

n n
i

(70)
1+i
= 2
i
Notice that the level payment perpetuity of Kl 1 has present value of
only 1i . Since i 0, we have i12 >> 1i .
HW: apply same analysis for decreasing annuities, with Km = n m + 1,
where m {1, 2, .., n}

Albert Cohen (MSU)

MATH 360: Theory of Investment and Credit

MSU Spring 2014

78 / 223

Example 2.22
Consider perpetuities X and Y with payment
Table: Example 2.22: Payments Schedule X

Week
Payment in

1
1

2
2

3
3

..n..
..n..

Table: Example 2.22: Payments Schedule Y

Week
Payment in

1
q

2
q

3
2q

4
2q

..2k 1
..kq

2k..
kq..

at the end of each year. It is known that NPV (X ) = NPV (Y ) when


valued at annual interest rate i = 0.10. Solve for q.
Albert Cohen (MSU)

MATH 360: Theory of Investment and Credit

MSU Spring 2014

79 / 223

Example 2.22
If we take eq := 2 =


1 2
1.1

1
1.21 ,

and so ieq = 0.21. It follows that

1+i
1.1
=
= 110
2
i
0.01
NPV (Y ) = q + q 2 + 2q 3 + 2q 4 + 3q 5 + 3q 6 + ...


= q + 2 3 + 3 5 + .. + q 2 + 2 4 + 3 6 + ..


q
= q+
2 + 2 4 + 3 6 + ..



q
2
3
= q+
+ 3eq
+ ..
eq + 2eq

1 + ieq
1.21
= 2.1 q
= 2.1 q
= 57.62q
2
ieq
0.212
NPV (X ) =

Albert Cohen (MSU)

MATH 360: Theory of Investment and Credit

MSU Spring 2014

80 / 223

Example 2.22
If we take eq := 2 =


1 2
1.1

1
1.21 ,

and so ieq = 0.21. It follows that

1+i
1.1
=
= 110
2
i
0.01
NPV (Y ) = q + q 2 + 2q 3 + 2q 4 + 3q 5 + 3q 6 + ...


= q + 2 3 + 3 5 + .. + q 2 + 2 4 + 3 6 + ..


q
= q+
2 + 2 4 + 3 6 + ..



q
2
3
= q+
+ 3eq
+ ..
eq + 2eq

1 + ieq
1.21
= 2.1 q
= 2.1 q
= 57.62q
2
ieq
0.212
NPV (X ) =

(71)

q = 1.91

Albert Cohen (MSU)

MATH 360: Theory of Investment and Credit

MSU Spring 2014

80 / 223

Continuous Payment Schemes


What if the payments Km were just discrete samples at time tm of a
continuous payment scheme? For example, the payment at time t = t1
was Ki = h(ti )dt.

Albert Cohen (MSU)

MATH 360: Theory of Investment and Credit

MSU Spring 2014

81 / 223

Continuous Payment Schemes


What if the payments Km were just discrete samples at time tm of a
continuous payment scheme? For example, the payment at time t = t1
was Ki = h(ti )dt.
Then all payments must be summed to obtain the NPV and AccValue:
Z n
1
NPV =
h(t)dt
(1
+
i)t
Z0 n
(72)
nt
AccVal =
(1 + i) h(t)dt
0

A special case is when h(t) = t. Then


Z n

1
I a
=
tdt
ni
(1 + i)t
Z0 n

I s
=
(1 + i)nt tdt
ni

(73)

Albert Cohen (MSU)

MATH 360: Theory of Investment and Credit

MSU Spring 2014

81 / 223

Continuous Payment Schemes


Integration by parts allows us to obtain
Z n

1
an i n n
I a
=
tdt
=
ni
(1 + i)t

Z0 n

sn i n
I s
=
(1 + i)nt tdt =
ni

0
:= ln (1 + i)

(74)

where is the constant force of interest.

Albert Cohen (MSU)

MATH 360: Theory of Investment and Credit

MSU Spring 2014

82 / 223

Continuous Payment Schemes


Integration by parts allows us to obtain
Z n

1
an i n n
I a
=
tdt
=
ni
(1 + i)t

Z0 n

sn i n
I s
=
(1 + i)nt tdt =
ni

0
:= ln (1 + i)

(74)

where is the constant force of interest.


If the force of interest varies, then
Z n R

t
I a
=
e 0 s ds tdt
n
Z0 n R

n
I s
=
e t s ds tdt
n

(75)

Albert Cohen (MSU)

MATH 360: Theory of Investment and Credit

MSU Spring 2014

82 / 223

Unique Interest Rates for Annuity Valuation: An


application of the Intermediate Value Theorem

Returning to the discrete setting, consider the following important theorem


Theorem
Assume that an annuity X pays amount Kj > 0 at time tj for all times
0 < t1 < t2 .. < tn . Suppose that you are P
given L > 0. Then there exists a
Kl
unique i > 1 such that NPV (X )(i) := nl=1 (1+i)
tl = L under rate i.
Without this theorem, we would not be able to uniquely price returns on
assets.

Albert Cohen (MSU)

MATH 360: Theory of Investment and Credit

MSU Spring 2014

83 / 223

Unique Interest Rates for Annuity Valuation: An


application of the Intermediate Value Theorem
Proof.
It follows that since {(Kl , tl )}nl=1 R2+ , we have NPV (X )0 (i) < 0. This
also follows from financial reasoning as a higher rate i would demand a
lower up front payment L. Since
lim NPV (X )(i) = 0

(76)

lim NPV (X )(i) =

i1

it follows from the IVT that there exists a unique i (due to strict
decreasing nature of NPV (X )(i)) such that NPV (X )(i) = L

Albert Cohen (MSU)

MATH 360: Theory of Investment and Credit

MSU Spring 2014

84 / 223

Yield and Reinvestment Rates

Assume a loan amount L for a term of n years, and a total value M of all
payments after the n years. Then it must be that for level payments K the
yield rate i satisfies
L=

K
K
K
M
=
+
+ .. +
= K an i
n
2
(1 + i)
1+i
(1 + i)
(1 + i)n

(77)

On a loan, the Internal Rate of Return, or IRR, is the rate of interest for
which the loan amount upront is equal in value to the NPV of all loan
payments. Also known as the loan rate. Sometimes, the entity that loans
L can reinvest these loan repayments at a higher rate.

Albert Cohen (MSU)

MATH 360: Theory of Investment and Credit

MSU Spring 2014

85 / 223

Ex. 2.25

(12)

Smith owns a 10000 savings bond that pays iBond = 0.06. Upon receipt of
an interest payment, he immediately deposits it into an account earning
(12)
interest, payable monthly, at a rate of iX = 0.12. Find the accumulated
value of this account just after the 12th , 24th , and 36th deposit. In each
(12)
case, find the average annual yield iavg based on his initial investment of
10000. Assume that the savings bond may be cashed in any time for
10000.

Albert Cohen (MSU)

MATH 360: Theory of Investment and Credit

MSU Spring 2014

86 / 223

Ex. 2.25

(12)

The interest paid each month is at a rate of i 12 = 0.06


12 = 0.5%. On an
investment of 10000, this means payments of 50 each month. The value
of his reinvestment of these payments at rate 0.01 is
Table: Example 2.25: Accumulated Value of Reinvestment

Month
Acc Val
=

Albert Cohen (MSU)

12
50 s12 0.01
634.13

24
50 s24 0.01
1348.67

36
50 s36 0.01
2153.84

MATH 360: Theory of Investment and Credit

MSU Spring 2014

87 / 223

Ex. 2.25

After 12 months, the equivalent rate j needed to accumulate


10000 + 50 s12 0.01 = 10634.13 satisfies
(12)

iavg 12
10000 1 +
= 10634.13
12
(12)
iavg = 0.0616

(78)

Do this for the other cases.

Albert Cohen (MSU)

MATH 360: Theory of Investment and Credit

MSU Spring 2014

88 / 223

Ex. 2.25

In general, we have
(12)

iavg (n) n
= 10000[1 + 0.005sn 0.01 ]
10000 1 +
12
h
(1.01)n 1 i
= 10000 1 + 0.005
0.01
1
h 1 1
i
n
(12)
iavg (n) = 12
+ (1.01)n 1
2 2

Albert Cohen (MSU)

MATH 360: Theory of Investment and Credit

MSU Spring 2014

(79)

89 / 223

Sinking Fund Method of Valuation

It often happens that interest payments returned to an investor at rate i


can only be reinvested at market rate j. If j 6= i, then the yield rate to
value the investment is also 6= i and we consider another way to determine
the value. Instead of using the present value method, we now consider the
value at the end of the term n of the investment.
When an investment is made, it can be considered a loan (principal) to the
person receiving the money up front. By the end of the term, the entire
principal has been repaid and the investor has enjoyed (consumed) periodic
interest payments for making the initial investment.
Consider, for example, the fair purchase price P of an annuity with level
payments K over n periods.

Albert Cohen (MSU)

MATH 360: Theory of Investment and Credit

MSU Spring 2014

90 / 223

Sinking Fund Method of Valuation


Receive periodic payments K in return for initial investment P.
The interest payment is P i, received every unit of time over interval
of length n.
These interest payments are consumed, and the excess K P i every
period is reinvested at a rate j into a sinking fund.
The fair value P is now determined via the recursive equation that
says investor recovers P at end of term, and this is equivalent to the
amount accumulated by reinvesting the excess K P i per period:
P = (K P i) sn j

Albert Cohen (MSU)

MATH 360: Theory of Investment and Credit

MSU Spring 2014

91 / 223

Sinking Fund Method of Valuation


Receive periodic payments K in return for initial investment P.
The interest payment is P i, received every unit of time over interval
of length n.
These interest payments are consumed, and the excess K P i every
period is reinvested at a rate j into a sinking fund.
The fair value P is now determined via the recursive equation that
says investor recovers P at end of term, and this is equivalent to the
amount accumulated by reinvesting the excess K P i per period:
P = (K P i) sn j
Ksn j
P=
1 + i sn j

Albert Cohen (MSU)

MATH 360: Theory of Investment and Credit

MSU Spring 2014

91 / 223

Sinking Fund Method of Valuation


Receive periodic payments K in return for initial investment P.
The interest payment is P i, received every unit of time over interval
of length n.
These interest payments are consumed, and the excess K P i every
period is reinvested at a rate j into a sinking fund.
The fair value P is now determined via the recursive equation that
says investor recovers P at end of term, and this is equivalent to the
amount accumulated by reinvesting the excess K P i per period:
P = (K P i) sn j
Ksn j
Ksn j
P=
=
i
1 + i sn j
1 + j ((1 + j)n 1)

Albert Cohen (MSU)

MATH 360: Theory of Investment and Credit

MSU Spring 2014

91 / 223

Sinking Fund Method of Valuation


Receive periodic payments K in return for initial investment P.
The interest payment is P i, received every unit of time over interval
of length n.
These interest payments are consumed, and the excess K P i every
period is reinvested at a rate j into a sinking fund.
The fair value P is now determined via the recursive equation that
says investor recovers P at end of term, and this is equivalent to the
amount accumulated by reinvesting the excess K P i per period:
P = (K P i) sn j
Ksn j
Ksn j
P=
=
i
1 + i sn j
1 + j ((1 + j)n 1)
lim P =

ji
Albert Cohen (MSU)

(80)

Ksn i
= Kan i
1 + (1 + i)n 1
MATH 360: Theory of Investment and Credit

MSU Spring 2014

91 / 223

Amortization

When repaying a loan L with periodic payments Km at time m, we have


the equation of value
L=

n
X
m=1

Km
(1 + i)m

(81)

In this method, we track the outstanding balance on a loan, denoted by


OB(i) or sometimes OBi .

Albert Cohen (MSU)

MATH 360: Theory of Investment and Credit

MSU Spring 2014

92 / 223

Amortization

OB0 = L

Albert Cohen (MSU)

MATH 360: Theory of Investment and Credit

MSU Spring 2014

93 / 223

Amortization

OB0 = L
OB1 = L (1 + i) K1

Albert Cohen (MSU)

MATH 360: Theory of Investment and Credit

MSU Spring 2014

93 / 223

Amortization

OB0 = L
OB1 = L (1 + i) K1
OB2 = OB1 (1 + i) K2

Albert Cohen (MSU)

MATH 360: Theory of Investment and Credit

MSU Spring 2014

93 / 223

Amortization

OB0 = L
OB1 = L (1 + i) K1
OB2 = OB1 (1 + i) K2
OBt+1 = OBt (1 + i) Kt+1 = OBt PRt+1
PRt+1 = Kt+1 i OBt
It+1 = i OBt

Albert Cohen (MSU)

MATH 360: Theory of Investment and Credit

MSU Spring 2014

93 / 223

Amortization

OB0 = L
OB1 = L (1 + i) K1
OB2 = OB1 (1 + i) K2
OBt+1 = OBt (1 + i) Kt+1 = OBt PRt+1

(82)

PRt+1 = Kt+1 i OBt


It+1 = i OBt
OBn = 0 = OBn1 (1 + i) Kn
Also
total interest
and cash paid out are, respectively,
Pn note that
Pthe
Pn
n
I
and
K

I
.
t=1 t
t=1 t
t=1 t

Albert Cohen (MSU)

MATH 360: Theory of Investment and Credit

MSU Spring 2014

93 / 223

Retrospective and Prospective Forms

At time t, if we look backwards to balance out how the original loan


amount has grown and subtract form that our total payments, then
OBt = L (1 + i)t

t
X

Km (1 + i)tm

(83)

m=1

If we take the NPV viewpoint, then the outstanding balance today is


simply the present value of all future payments until the the loan is paid
off:
OBt =

n
X
m=t+1

Albert Cohen (MSU)

Km
(1 + i)mt

MATH 360: Theory of Investment and Credit

(84)

MSU Spring 2014

94 / 223

Example 3.2

A loan of 3000 at an effective quarterly rate of j = 0.02 is amortized by


means of 12 quarterly payments, beginning one quarter after the loan is
made. Each payment consists of a principal repayment of 250 plus interest
due on the previous quarters outstanding balance. Construct the
amortization schedule.

Albert Cohen (MSU)

MATH 360: Theory of Investment and Credit

MSU Spring 2014

95 / 223

Example 3.2

In this case, we have level princiapl repayments, and so PRt = 250 for all
0 t 12. Since It+1 = 0.02 OBt , it follows that
OBt+1 = OBt 250
OBt = 3000 250t
It = 0.02 (3000 250 (t 1)) = 65 5t

(85)

Kt = It + PRt = 250 + 65 5t = 315 5t

Albert Cohen (MSU)

MATH 360: Theory of Investment and Credit

MSU Spring 2014

96 / 223

Amortization of a Loan with Level Repayments

In this case, which is common, the payments are held constant, say at
K, but the proportion of interest paid versus principal paid varies with
time.
Examples such as mortgages, car payments, etc..
Can use retrospective method to value outstanding balance at time t.

Albert Cohen (MSU)

MATH 360: Theory of Investment and Credit

MSU Spring 2014

97 / 223

Amortization of a Loan with Level Repayments

Here, the outstanding balance


Pt is of course OBt = K ant i , and the total
cash paid out by time t is m=1 Kt = K t. It follows that, with
= ln (1 + i)

It = i OBt1 = i K ant+1 i = K 1 (1 + i)tn1
K
PRt = Kt It =
(86)
(1 + i)n+1t


K
OBt =
1 e (nt)
i

Albert Cohen (MSU)

MATH 360: Theory of Investment and Credit

MSU Spring 2014

98 / 223

Sinking Fund Method of Valuation Revisited

Even though the outstanding balance is paid off only at the end of the
term of the loan, we can still interpolate and say that the outstanding
balance decreases as
OBt = L

L
s
sn j t j

(87)

where the balance decreases by the amount paid into the account earning
at a rate j.

Albert Cohen (MSU)

MATH 360: Theory of Investment and Credit

MSU Spring 2014

99 / 223

Sinking Fund Method of Valuation Revisited

Note that we can also calculate the net interest payment each month via
Kt = L i +

L
sn j



st j st1 j
PRt = OBt1 OBt = L
sn j


st1 j
It = Kt PRt = L i j
sn j

Albert Cohen (MSU)

MATH 360: Theory of Investment and Credit

MSU Spring 2014

(88)

100 / 223

Sinking Fund Example from previous SOA Exam FM/1

Ronaldo borrows L for 10 years at an annual effective interest rate of


100 i%. At the end of each year, he pays the interest on the loan and
deposits the level amount necessary to repay the principal to a sinking
fund earning an annual effective interest rate of 100 j%. The total
payments made by Ronaldo over the 10-year period is X. Calculate X .

Albert Cohen (MSU)

MATH 360: Theory of Investment and Credit

MSU Spring 2014

101 / 223

Sinking Fund Example from previous SOA Exam FM/1

Answer:
Annual interest payment = L i
Annual sinking fund deposit is

L
s10|j

Total annual payment = L i + s L


10|j

Total of all payments = 10 L i +

Albert Cohen (MSU)

s10|j

MATH 360: Theory of Investment and Credit

MSU Spring 2014

102 / 223

Makehams Formula

Consider the following scenario: An investor makes a loan at interest rate


i, with only interest payments for the term of the loan, and then a lump
sum L at the end of term. The investor then sells the loan to a speculator,
who values the expected cash flows at a rate j. Then the value the
speculator puts on the investment is


i
L
L
L
+ L i an j =
+ L
(89)
A=
(1 + j)n
(1 + j)n j
(1 + j)n

Albert Cohen (MSU)

MATH 360: Theory of Investment and Credit

MSU Spring 2014

103 / 223

Makehams Formula

This idea generalizes to the P


case where the principal L is paid over a series
n
of lump
sum
payments
L
=
m=1 Lm and periodic repayments
Pn
K = m=1 Km .

Albert Cohen (MSU)

MATH 360: Theory of Investment and Credit

MSU Spring 2014

104 / 223

Makehams Formula

This idea generalizes to the P


case where the principal L is paid over a series
n
of lump
sum
payments
L
=
m=1 Lm and periodic repayments
Pn
K = m=1 Km .
Each of these lump sum payments can be viewed as corresponding to an
individual loan:


Ls
Ls
i
Ls
+ Ls i ats j =
+ Ls
As =
(1 + j)ts
(1 + j)ts
j
(1 + j)ts

Albert Cohen (MSU)

MATH 360: Theory of Investment and Credit

MSU Spring 2014

(90)

104 / 223

Makehams Formula

We can find the total value now by summing up the individual loan values
n
X

n
X



Ls
i
Ls

+
s
(1 + j)ts
j
(1 + j)ts
s=1
s=1
!
n
n
n
X
X
X
Ls
Ls
i
Ls
=
+
(1 + j)ts
j
(1 + j)ts

A=

As =

s=1

s=1

(91)

s=1

i
= K + (L K )
j

Albert Cohen (MSU)

MATH 360: Theory of Investment and Credit

MSU Spring 2014

105 / 223

Mortgage Repayment from previous SOA Exam FM/1

A loan L is being repaid with n annual level payments of K each. With the
mth payment, m < n, the borrower pays an extra amount A, and then
agrees to repay the remaining balance over l years with a revised annual
payment. The effective rate of interest is 100 i%. . Calculate the amount
of the revised annual payment.

Albert Cohen (MSU)

MATH 360: Theory of Investment and Credit

MSU Spring 2014

106 / 223

Mortgage Repayment from previous SOA Exam FM/1

With n m yealy payments left right before the extra repayment, the
remaining balance is K a(nm)|i . Right after the extra payment A, the
remaining balance is K a(nm)|i A. Hence, the new payment over the
new 10year schedule is
Knew =

Albert Cohen (MSU)

K a(nm)|i A
al|i

MATH 360: Theory of Investment and Credit

(92)

MSU Spring 2014

107 / 223

Bonds

A Bond is a financial contract where an up front payment is exchanged for


coupon payments (periodic interest paid to the debt-holder) plus a face
amount the maturity date (end of term). The Bond is valued using a yield
rate used to value future cash flows. The notation used with bond
valuation is
j = the Yield rate per period
F = the Face amount
r = the Coupon rate
C = the redemption amount, usually equal to the face value
N = the number of coupon payments until maturity
P = the price of the bond, where

Albert Cohen (MSU)

MATH 360: Theory of Investment and Credit

MSU Spring 2014

108 / 223

Bonds

The bond can now be valued as


F
+F r
P=
(1 + j)n
= F n + F r an j

1
1
+ .. +
1+j
(1 + j)n

(93)

= F + F (r j) an j
r
= F n + (F F n )
j
Note that Bond prices are usually listed, and the investor determines yield
rate from listed price. To do so, must invert our Bond pricing formula.
This requires some calculation.

Albert Cohen (MSU)

MATH 360: Theory of Investment and Credit

MSU Spring 2014

109 / 223

Example 4.1

Price the following bond with face 100:


Issue Date: March 1,2004
Maturity Date: Feb 28, 2006
r = 1.625%
j = 1.675%
Term is 2 years (4 six month periods)

Albert Cohen (MSU)

MATH 360: Theory of Investment and Credit

MSU Spring 2014

110 / 223

Example 4.1

Price the following bond with face 100:


Issue Date: March 1,2004
Maturity Date: Feb 28, 2006
r = 1.625%
j = 1.675%
Term is 2 years (4 six month periods)

P = 100 0.5 0.01625

1
1
+ ... +
1.008375
1.0083754

100
+
= 99.02
1.0083754

Albert Cohen (MSU)

MATH 360: Theory of Investment and Credit

MSU Spring 2014

(94)

110 / 223

Bonds

At time t, we define Pt to be the price of the bond, just after the coupon
payment. This is equal to the NPV of the remainin payments:
Pt =

F
+ F r ant j
(1 + j)nt

(95)

Now, since the initial price P satisfies the relation P F = F (r j)an j ,


we say that a bond is bought at premium if r > j, at par if r = j, and at a
discount if r < j.

Albert Cohen (MSU)

MATH 360: Theory of Investment and Credit

MSU Spring 2014

111 / 223

Bond Pricing Between Coupon Dates

Let
u :=

the number of days since last coupon payment


number of days in coupon period

(96)

Then Pt = Pm (1 + j)u u F r , where m is the number of coupon


payments left and Pm (1 + j)u is known as the price plus accrued.

Albert Cohen (MSU)

MATH 360: Theory of Investment and Credit

MSU Spring 2014

112 / 223

Amortization of Bond

Sometimes need to determine the Book Value of a bond for tax or investor
purposes. The Book Value at time t is the outstanding balance at time t
right after the coupon payment. This is the present value of the remaining
cash payments of that the bond holder is entitled to receive:
Pt = OBt = F + F (r j) ant j

Albert Cohen (MSU)

MATH 360: Theory of Investment and Credit

(97)

MSU Spring 2014

113 / 223

Amortization of Bond

We can use the idea that we are loaning money to bond-issuer, and that
we price this as a mortgage. So,
Pt = OBt = F + F (r j) ant j
Kt = F r for t {1, 2, 3, .., n 1}
Kn = F r + F

(98)

It = Kt PRt
PRt = Kt (OBt1 OBt )
= F (r j) nt+1

Albert Cohen (MSU)

MATH 360: Theory of Investment and Credit

MSU Spring 2014

114 / 223

Bond Pricing Lemma

Lemma
Consider a bond with face F , coupon rate r , and term T = 2n, n N,
purchased by A for a price PA . Just after the nth coupon, A sells the bond
to B who desires a yield j and correspondingly pays PB . If PB > PA and
j < r or
j > r and PB >

F +PA
2

then i > j, where i is the non-zero yield rate calculated on A0 s investment.

Albert Cohen (MSU)

MATH 360: Theory of Investment and Credit

MSU Spring 2014

115 / 223

Bond Pricing Lemma: Proof 1


Proof.
We prove each case separately, both of which reveal something about the
situation described above.
Case 1 First, if j < r
PB
PA = F r an i +
(1 + i)n


Fr
1
PA
>
1
+
n
i
(1 + i)
(1 + i)n
PB = F + F (r j) an j

(99)

We can see that this implies


Fr
r j
Fr
< PA < PB = F + F (r j) an j < F + F
=
i
j
j

(100)

and so i > j
Albert Cohen (MSU)

MATH 360: Theory of Investment and Credit

MSU Spring 2014

116 / 223

Bond Pricing Lemma: Proof 2


Proof.
Case 2 If j > r then by Makehams formula F > PB > PA and
PB
(1 + i)n


PB F
F
+
+
(1 + i)n
(1 + i)n
F
+
(1 + j)n

PA = F ran i +
= F ran i
PB = F ran j

(101)

For the sake of contradiction, assume i j. Then it follows by our


assumptions that
F PB < PB PA

F PB
(1 + i)n

(102)

which is impossible.
Albert Cohen (MSU)

MATH 360: Theory of Investment and Credit

MSU Spring 2014

117 / 223

Exotic Bonds

Callable Bonds: Bond can be recalled by issuer over a window of


redemption dates. Price investor pays is the minimum of these possible
prices. So, if the bond was purchased at a premium, then price at earliest
possible date. If purchased at a discount, price at latest possible
redemption date

Albert Cohen (MSU)

MATH 360: Theory of Investment and Credit

MSU Spring 2014

118 / 223

Exotic Bonds

Serial Bonds: Face amount F is redeemed over a collection of dates


{t1 , .., tn } with staggered payments F1 + .. + Fn = F , coupon rates
r1 , .., rn , tailored to yield rates j1 , .., jn . One can view this as the sum of m
bonds with the corresponding payments, and can use Makehams formula
to price this
P=

n
X

Pm

m=1

(103)



rt
Ft
Ft
+ Ft
Pt =
(1 + j)nt
jt
(1 + jt )nt

Albert Cohen (MSU)

MATH 360: Theory of Investment and Credit

MSU Spring 2014

119 / 223

Measuring The Rate of Return Of An Investment

Yield rate can be derived from cash-flows from an investment


When presented with more than one investment opportunity, would
like to be able to quantitatively measure their value to the investor
There are many ways to do this some work in situations that others
dont

Albert Cohen (MSU)

MATH 360: Theory of Investment and Credit

MSU Spring 2014

120 / 223

Internal Rate of Return

In an investment, at any time may have cash flows out and in say
payments to buy a license for a taxicab, with monthly maintenance fees
versus fare payments in.
Notation is such that Ak is a payment received and Bk is a payment made
out, each at the same time tk . The net payment is thus Ck .

f (j) :=

n
X
k=1

Ck
(1 + j)tk

(104)

The Internal Rate of Return j is one that balances out all net payments so
that their net present value is 0.

Albert Cohen (MSU)

MATH 360: Theory of Investment and Credit

MSU Spring 2014

121 / 223

Example 5.1

Smith buys 1000 shares of stock at 5.00 per share and pays a commission
of 2%. Six months later he receives a cash-dividend of 0.20 per share,
which he immediately reinvests commission free in shares at a price of 4.00
per share. Six months after that he buys another 500 shares at a price
4.50 per share, along with a commission of 2%. Six months after that he
receives another cash dividend of 0.25 per share and sells his existing
shares at 5.00 per share, again paying a 2% commission. Find Smiths
internal rate of return for the entire transaction in the form i (2)

Albert Cohen (MSU)

MATH 360: Theory of Investment and Credit

MSU Spring 2014

122 / 223

Example 5.1
Let 0 represent the time of the original share purchase, t = 1 at 6 months,
t = 2 at 12 months and t = 3 at 18 months after the original purchase.
Then
(A0 , B0 ) = (0, 5000 + [0.02 5000]) = (0, 5100)
(A1 , B1 ) = (200, 200)
(A2 , B2 ) = (0, [1.02 4.50 500]) = (0, 2295)
(A3 , B3 ) = ([0.25 + 0.98 5] 1550, 0)

(105)

= (7982.50, 0)
(C0 , C1 , C2 , C3 ) = (5100, 0, 2295, 7982.5)
0 = 5100 + 0 2295 2 + 7982.50 3
Solving this polynomial, we obtain
=
Albert Cohen (MSU)

1
i (2) = 2j = 0.0649
1+j

MATH 360: Theory of Investment and Credit

(106)
MSU Spring 2014

123 / 223

Uniqueness? Guaranteed Solution?

If C0 < 0 and Cj > 0 for all j {1, 2, ..n}, then by the Intermediate
Value theorem and simple calculus, we have a unique solution for the
internal return rate j. (i.e. Mortgages or other loans paid off by
periodic payments)
BUT, we can have situations where the above isnt satisfied, and we
compute more than solution for j, or worse, no real solution.
What can we use to measure the value of an investment in this case?

Albert Cohen (MSU)

MATH 360: Theory of Investment and Credit

MSU Spring 2014

124 / 223

Net Present Value

Assume that among all possible investment alternatives, all have same
measure of risk and fixed interest rate i that investor proposes
(Utility theory!)
In this incomplete case, use i, also known as cost of capital, to find
present value of cash flows of each investment on the table
The one with the largest present value, given investors rate i, is most
preferable. In symbols, for investments a, b, c, with NPV
fa (i), fb (i), fc (i), choose the biggest
HW: Can NPV curves ever intersect for different values of i ?
Of course, if i = j =yield rate, then NPV = 0

Albert Cohen (MSU)

MATH 360: Theory of Investment and Credit

MSU Spring 2014

125 / 223

Profitability Index

Again, if the investor proposes his own interest rate i, then we can use
another measure, the Profitability Index:
I =

PV [ Cash Flows In]


PV [ Cash Flows Out ]

(107)

Note that if i = j = IRR, then I = 1 by design,as yield rate is what we use


to balance out flows with in flows.

Albert Cohen (MSU)

MATH 360: Theory of Investment and Credit

MSU Spring 2014

126 / 223

Profitability Index: Example

Compare
(a) lending 1000 and being repaid 250 per year for 5 years with
i = 0.05 and
(b) lending 1000 and being repaid 140 per year for 10 years with
i = 0.05

Albert Cohen (MSU)

MATH 360: Theory of Investment and Credit

MSU Spring 2014

127 / 223

Profitability Index: Example

Compare
(a) lending 1000 and being repaid 250 per year for 5 years with
i = 0.05 and
(b) lending 1000 and being repaid 140 per year for 10 years with
i = 0.05
We compute
250a5 0.05
= 1.0824
1000
140a10 0.05
Ib =
= 1.0810
1000
Ia =

Albert Cohen (MSU)

MATH 360: Theory of Investment and Credit

(108)

MSU Spring 2014

127 / 223

MIRR

Goal: Find j such that Accumulated Value at end of project of payments


out at rate j matches Accumulated Value of payments received at
investors cost of capital i.
Notation:
Ak is the payment in at time k
Bk is the payment out at time k

Albert Cohen (MSU)

MATH 360: Theory of Investment and Credit

MSU Spring 2014

128 / 223

MIRR

Goal: Find j such that Accumulated Value at end of project of payments


out at rate j matches Accumulated Value of payments received at
investors cost of capital i.
Notation:
Ak is the payment in at time k
Bk is the payment out at time k
Then j solves the equation
N
X
n=0

Albert Cohen (MSU)

An (1 + i) =

N
X

Bm (1 + j)m

(109)

m=0

MATH 360: Theory of Investment and Credit

MSU Spring 2014

128 / 223

MIRR: Example
Compare
(a) lending 1000 and being repaid 250 per year for 5 years with
i = 0.05 and
(b) lending 1000 and being repaid 140 per year for 10 years with
i = 0.05

Albert Cohen (MSU)

MATH 360: Theory of Investment and Credit

MSU Spring 2014

129 / 223

MIRR: Example
Compare
(a) lending 1000 and being repaid 250 per year for 5 years with
i = 0.05 and
(b) lending 1000 and being repaid 140 per year for 10 years with
i = 0.05
We compute
1000(1 + ja )5 = 250s5 0.05 = 1381.41
ja = 0.0668
1000(1 + jb )10 = 140s10 0.05 = 1760.90

(110)

jb = 0.0582

Albert Cohen (MSU)

MATH 360: Theory of Investment and Credit

MSU Spring 2014

129 / 223

Project Return Rate


There are times during a project where investor is borrower and times
when she is lender. Cost of financing at rate i, and project return rate is j
when she is net lender. Solving j in terms of i would give minimum return
rate needed for project to break even if set NPV to 0 at end of project.
Goal: Set up an equation of value such that Net Project Value = 0 when
t = N. If the value at t = n is > 0, then earn at rate j. If balance < 0,
then earn (pay) at rate i (cost of capital). As an example, consider
S0 = 1 S1 = (1 + j)
S1 = 1 + j 2.3 S2 = (1 + j 2.3)(1 + i)
(111)

S2 = (1 + j 2.3)(1 + i) + 1.33 = 0
1.33
j = 1.3
1+i

Albert Cohen (MSU)

MATH 360: Theory of Investment and Credit

MSU Spring 2014

130 / 223

Dollar Weighted Rate of Return

Goal: Calculate measure of return over 1 year.

Albert Cohen (MSU)

MATH 360: Theory of Investment and Credit

MSU Spring 2014

131 / 223

Dollar Weighted Rate of Return

Goal: Calculate measure of return over 1 year. Use Simple Interest for
fraction of years. Define interest gained as the difference between the total
amount paid out during year to term and the total amount paid in during
year to term.
idollar weigted =

Albert Cohen (MSU)

Total Interest Gained


Average Amount on deposit during year

MATH 360: Theory of Investment and Credit

MSU Spring 2014

(112)

131 / 223

Example 5.3

A Pension fund receives contributions and pays benefits from time to time.
The fund began the year 2005 with a balance of 1, 000, 000. There were
contributions to the fund of 200, 000 at the end of February and again at
the end of August. There was a benefit of 500, 000 paid out of the fund at
the end of October. The balance remaining at the start of the year 2006
was 1, 100, 000. Find the dollar weighted return on the fund, assuming
1
each month is 12
of a year

Albert Cohen (MSU)

MATH 360: Theory of Investment and Credit

MSU Spring 2014

132 / 223

Example 5.3



10
1, 100, 000 = 1, 000, 000(1 + i) + 200, 000 1 + i
12




2
4
+ 200, 000 1 + i 500, 000 1 + i
12
12
T .I .G . = 1, 100, 000 + 500, 000 1, 000, 000
200, 000 200, 000 = 200, 000
10
A.A.o.d.d.y = 1, 000, 000 + 200, 000
12
4
2
+ 200, 000
500, 000
= 1, 150, 000
12
12
200, 000
i =
= 0.1739
1, 150, 000

Albert Cohen (MSU)

MATH 360: Theory of Investment and Credit

MSU Spring 2014

(113)

133 / 223

Even More Methods of Valuation

Time Weighted Rate of Return: Compound over fraction of years, but


the time length doesnt matter
Portfolio Method: Contributions to a fund, once opened, are
segregated from initial investment and accrue interest at a different
rate for a fixed term
Interest Preference Rates for Borrowing and Lending: If accumulated
amount falls below zero, pay interest at a different rate than interest
credited on a positive balance. This is useful to compare two
alternatives when one may have a negative balance for some time

Albert Cohen (MSU)

MATH 360: Theory of Investment and Credit

MSU Spring 2014

134 / 223

Continuous Withdrawal and Investment


Begin with the equation for accumulated value

F (t2 ) = F (t1 )(1+i)t2 t1 +

n
X
k=1

Albert Cohen (MSU)

ctk (1+i)t2 tk +

t2

c(t)(1+i)t2 t dt (114)

t1

MATH 360: Theory of Investment and Credit

MSU Spring 2014

135 / 223

Continuous Withdrawal and Investment


Begin with the equation for accumulated value

F (t2 ) = F (t1 )(1+i)t2 t1 +

n
X

ctk (1+i)t2 tk +

k=1

t2

c(t)(1+i)t2 t dt (114)

t1

As an example, consider t1 = 0, t2 = 1, tk = kn , ctk = 1, c(t) = N n.


Then
Z 1
n
X
1 kn
F (1) = F (0) (1 + i) +
(1 + i)
+ (N n)
(1 + i)1t dt
0

k=1

(115)

(N n) i
= F (0) (1 + i) +
+
1
ln (1 + i)
(1 + i) n 1
i

Albert Cohen (MSU)

MATH 360: Theory of Investment and Credit

MSU Spring 2014

135 / 223

Continuous Withdrawal and Investment


Begin with the equation for accumulated value

F (t2 ) = F (t1 )(1+i)t2 t1 +

n
X

ctk (1+i)t2 tk +

k=1

t2

c(t)(1+i)t2 t dt (114)

t1

As an example, consider t1 = 0, t2 = 1, tk = kn , ctk = 1, c(t) = N n.


Then
Z 1
n
X
1 kn
F (1) = F (0) (1 + i) +
(1 + i)
+ (N n)
(1 + i)1t dt
0

k=1

(115)

(N n) i
= F (0) (1 + i) +
+
1
ln (1 + i)
(1 + i) n 1
i

Now, given F (0), F (1), n, N, solve for i.


Albert Cohen (MSU)

MATH 360: Theory of Investment and Credit

MSU Spring 2014

135 / 223

Term Structure of Interest Rates

The relationship of the yield of a bond (or other fixed income


product) to its time to maturity is the Term Structure
The graph of this relationship is the Yield Curve
Term Structure is dynamic ; reflects new information gained as
markets evolve
A yield curve that corresponds to Normal Term Structure is one that
sees yield increase when the time to maturity increases. When the
yield decreases as term increases, we say the yield curve is Inverted,
and if it is roughly constant, we say it is a Flat curve.

Albert Cohen (MSU)

MATH 360: Theory of Investment and Credit

MSU Spring 2014

136 / 223

Law of One Price

Bond Prices are linear. If we have a coupon paying bond, it must


have the same value as the sum of the prices of each coupon as an
individual zero coupon bond, as well as the lump sum paid at
maturity.
If this does not hold, then we have observed a case of arbirtrage. This
means one could construct a portfolio that consists of purchasing
individual pieces of a coupon bond while selling the equivalent whole
bond, making a net profit with no risk involved.

Albert Cohen (MSU)

MATH 360: Theory of Investment and Credit

MSU Spring 2014

137 / 223

Example 6.1
Suppose that the current term structure has the following yields on
zero-coupon bonds, where all yields are nominal annual rates of interest
compounded semi-annually
Table: Example 6.1: Term Structure

Term (Years)
Zero Coupon Bond Rate

0.5
8%

1
9%

1.5
10%

2
11%

Find the price per 100 face amount and yield to maturity of each of the
following 2-year bonds (with semi-annual coupons)
a.) Zero coupon bond
b.) 5% annual coupon rate
c.) 10% annual coupon rate
Albert Cohen (MSU)

MATH 360: Theory of Investment and Credit

MSU Spring 2014

138 / 223

Example 6.1

a.) PV =

100

(1+ 0.11
2 )

= 80.72, which implies j (2) = 0.11

b.)
2.5
2.5
2.5
102.5
+
+
+
= 89.59
2
3
1.04 1.045
1.05
1.0554
2.5
2.5
2.5
102.5
=
+
2 + 
3 + 
 (116)
j (2)
(2)
(2)
(2) 4
1+ 2
1 + j2
1 + j2
1 + j2

PV =

j (2) = 0.109354

Albert Cohen (MSU)

MATH 360: Theory of Investment and Credit

MSU Spring 2014

139 / 223

Example 6.1

c.) Similarly,
5
5
5
105
+
+
+
2
3
1.04 1.045
1.05
1.0554
= 98.46
5
5
105
5
+
=
2 + 
3 + 

j (2)
(2)
(2)
(2) 4
1+ 2
1 + j2
1 + j2
1 + j2

PV =

(117)

and so j (2) = 0.108775

Albert Cohen (MSU)

MATH 360: Theory of Investment and Credit

MSU Spring 2014

140 / 223

Spot Rates

The Spot Rate st is the effective annual interest rate for a zero
coupon bond maturing t years from now.
There is a relationship between the spot rate and the yield to
maturity of a bond.
If term structure of spot rates is increasing, then yield for bonds with
same maturity decreases as coupons increase

Albert Cohen (MSU)

MATH 360: Theory of Investment and Credit

MSU Spring 2014

141 / 223

Spot Rates and Yield Rates

The general equation of value here to determine the yield rate yr on a


bond with face 1 is

1
1
1
1
+
+ .. +
+
PV = r
2
n
1 + s1 (1 + s2 )
(1 + sn )
(1 + sn )n


1
1
1
1
=r
+
+ .. +
+
2
n
1 + yr
(1 + yr )
(1 + yr )
(1 + yr )n


Albert Cohen (MSU)

MATH 360: Theory of Investment and Credit

MSU Spring 2014

(118)

142 / 223

Spot Rates and Yield Rates

Solving for r , we obtain


1
(1+yr )n
1
k=1 (1+yr )k

r = Pn

Albert Cohen (MSU)

1
(1+s )n

Pn n

(119)

1
k=1 (1+sk )k

MATH 360: Theory of Investment and Credit

MSU Spring 2014

143 / 223

Spot Rates and Yield Rates

Q: Is there anything we can say about

Albert Cohen (MSU)

dyr
dr

given a normal term structure?

MATH 360: Theory of Investment and Credit

MSU Spring 2014

144 / 223

Spot Rates and Yield Rates

Q: Is there anything we can say about


A: Implicit differentiation leads to

dyr
dr

given a normal term structure?

1
1
dyr
1
(1+sn )n (1+yr )n
= Pn
k
dr
r r k=1
+ (1+ynr )n+1
(1+yr )k+1

(120)

<0
if we have a normal term structure as we expect sn > yr and so
1
1
(1+sn )n < (1+yr )n .

Albert Cohen (MSU)

MATH 360: Theory of Investment and Credit

MSU Spring 2014

144 / 223

Forward Rates

In keeping with Law of One Price (No Arbitrage), can set up the
relationship between spot rates and forward rates
Forward Rate is the interest rate to be charged starting at time t for a
fixed period
We will talk about 1year Forward Rates, i.e. interest rate charged
from to t + 1

Albert Cohen (MSU)

MATH 360: Theory of Investment and Credit

MSU Spring 2014

145 / 223

Exact Form

If we set up a deal such that 1 is to be invested at time 0, and the 1year


spot rate s1 = 0.08 , and the 2year spot rate is s2 = 0.09, then at t = 1,
if the rate to borrow and lend is the same, the rate from year 1 to year 2,
i1,2 , should be defined via
1.092 = (1 + i1,2 ) 1.08

(121)

i1,2 = 0.1001

Albert Cohen (MSU)

MATH 360: Theory of Investment and Credit

MSU Spring 2014

146 / 223

General Relationship between Spot Rates and Forward


Rates

1 + s1 = 1 + i0,1
(1 + s2 )2 = (1 + i0,1 )(1 + i1,2 )
.. ..
.=.

(122)

(1 + sn )n = nk=1 (1 + ik1,k )
Inverting this relationship, we obtain
ik1,k =

Albert Cohen (MSU)

(1 + sk )k
1
(1 + sk1 )k1

MATH 360: Theory of Investment and Credit

(123)

MSU Spring 2014

147 / 223

General Relationship between Spot Rates and Forward


Rates

1 + s1 = 1 + i0,1
(1 + s2 )2 = (1 + i0,1 )(1 + i1,2 )
.. ..
.=.

(122)

(1 + sn )n = nk=1 (1 + ik1,k )
Inverting this relationship, we obtain
ik1,k =

(1 + sk )k
1
(1 + sk1 )k1

(123)

Q: For a normal term structure where sk > sk1 , what can we say about
the relationship between sk and ik1,k ?

Albert Cohen (MSU)

MATH 360: Theory of Investment and Credit

MSU Spring 2014

147 / 223

General Relationship between Spot Rates and Forward


Rates

From the previous slide, we can compute the following table:


Table: Example 6.3: Term Structure

Spot Rate
Forward Rate

Albert Cohen (MSU)

s1 = 0.05
i0,1 = 0.05

s2 = 0.1
i1,2 = 0.1524

MATH 360: Theory of Investment and Credit

s3 = 0.15
i2,3 = 0.2596

MSU Spring 2014

148 / 223

Force of Interest vs YTM

Define t as the continuously compounded yield rate. Then


PV (t = 0) = Fe tt
Rt

= Fe 0 s ds
Z t
tt =
s ds

(124)

dt
= t
t + t
dt

Albert Cohen (MSU)

MATH 360: Theory of Investment and Credit

MSU Spring 2014

149 / 223

Force of Interest vs YTM

Summarizing, we have the dual relationship


Z
1 t
s ds
t =
t 0
dt
t = t + t
dt

Albert Cohen (MSU)

MATH 360: Theory of Investment and Credit

(125)

MSU Spring 2014

150 / 223

Force of Interest vs YTM

Recall that in the discrete case,


1

1 + sn = [(i + i0,1 ) (1 + i1,2 ) ... (1 + in1,n )] n

Albert Cohen (MSU)

MATH 360: Theory of Investment and Credit

MSU Spring 2014

(126)

151 / 223

Force of Interest vs YTM

Recall that in the discrete case,


1

1 + sn = [(i + i0,1 ) (1 + i1,2 ) ... (1 + in1,n )] n

(126)

Hence, 1 + sn is the geometric mean of the forward rates.

Albert Cohen (MSU)

MATH 360: Theory of Investment and Credit

MSU Spring 2014

151 / 223

Force of Interest vs YTM

Recall that in the discrete case,


1

1 + sn = [(i + i0,1 ) (1 + i1,2 ) ... (1 + in1,n )] n

(126)

Hence, 1 + sn is the geometric mean of the forward rates.


In the continuous case, the YTM t is the arithmetic mean of the force of
interest over the interval [0, t]. It follows that t can be seen as a forward
rate in the continuous case.

Albert Cohen (MSU)

MATH 360: Theory of Investment and Credit

MSU Spring 2014

151 / 223

Force of Interest vs YTM

Recall that in the discrete case,


1

1 + sn = [(i + i0,1 ) (1 + i1,2 ) ... (1 + in1,n )] n

(126)

Hence, 1 + sn is the geometric mean of the forward rates.


In the continuous case, the YTM t is the arithmetic mean of the force of
interest over the interval [0, t]. It follows that t can be seen as a forward
rate in the continuous case.
Q: Again, if the term structure is increasing, ie. if
t relate to t ?

Albert Cohen (MSU)

MATH 360: Theory of Investment and Credit

dt
dt

> 0, then how does

MSU Spring 2014

151 / 223

Example 6.5

Suppose that the yield to maturity for a zero-coupon bond maturing at


time t is t = 0.09 0.08 0.94t , a continuously compounded rate.
Forward Rate is the interest rate to be charged starting at time t for a
fixed period
(a) Find the related forward rate
(b) A borrower plans to borrow 1000 in one year and repay the loan
with a single payment at the end of the second year. Determine the
amount that will have to be paid back based on the stated term
structure

Albert Cohen (MSU)

MATH 360: Theory of Investment and Credit

MSU Spring 2014

152 / 223

Example 6.5

a.) We find that


t = t + t

dt
= 0.09 0.08 0.94t 0.08 0.94t ln (0.94) t (127)
dt

b.) We find that


Value = 1000e

Albert Cohen (MSU)

R2
1

t dt

= 1024.11

MATH 360: Theory of Investment and Credit

MSU Spring 2014

153 / 223

Example 6.5

a.) We find that


t = t + t

dt
= 0.09 0.08 0.94t 0.08 0.94t ln (0.94) t (127)
dt

b.) We find that


Value = 1000e

Albert Cohen (MSU)

R2
1

t dt

= 1024.11 = 1000e 22 1

MATH 360: Theory of Investment and Credit

MSU Spring 2014

(128)

153 / 223

At Par Yield

Measure of Bond Yield: Find the Coupon rate rt such that the Bond
Yield is also rt .
In this case, the Bond price is at par.

Albert Cohen (MSU)

MATH 360: Theory of Investment and Credit

MSU Spring 2014

154 / 223

At Par Yield

Measure of Bond Yield: Find the Coupon rate rt such that the Bond
Yield is also rt .
In this case, the Bond price is at par.
For an nyear bond,
1 = rn
rn =

Albert Cohen (MSU)

n
X
(1 + sk )k

!
+

k=1
1 (1+s1 n )n
Pn
1
k=1 (1+sk )k

MATH 360: Theory of Investment and Credit

1
(1 + sn )n
(129)

MSU Spring 2014

154 / 223

Interest Rate Swaps

Used to convert Floating Rate Liability to Fixed Rate Liability


Used between two borrowers that have access to different interest
markets
Example
(FixedRate(A), FloatingRate(A)) = (0.08, Prime + 0.01)
(FixedRate(B), FloatingRate(B)) = (0.09, Prime + 0.015)

Albert Cohen (MSU)

MATH 360: Theory of Investment and Credit

MSU Spring 2014

(130)

155 / 223

Interest Rate Swaps

Swap: A borrows at FixedRate(A), and loans to B at rate 0.085. B


borrows at FloatingRate(B), and loans to A at rate prime + 0.0125.
Net effect:
A pays (0.08 0.085 + prime + 0.0125) = prime + 0.0075
B pays (prime + 0.015 (prime + 0.0125) + 0.085) = 0.0875
Notice both of these are smaller than what they could have gotten on their
own. Usually, there is a question of default risk, so an intermediary can be
brought in, for a small fee, depending on the spread between available
rates for A and B

Albert Cohen (MSU)

MATH 360: Theory of Investment and Credit

MSU Spring 2014

156 / 223

Interest Rate Swaps

In general, a swap is a series of coupons exchanged upon delivery.


Imagine a bond with face value F , and term structure (implied
forward rates) for period k 1 to k: {ik1,k }nk=1 .
Then the implied swap rate Rn for term n is the rate that equates
the present value all the coupon exchanges, forward rate to constant
Rn , to 0:

0=

n
X
(Rn ik1,k )F
k=1

Albert Cohen (MSU)

(131)

(1 + sk )k

MATH 360: Theory of Investment and Credit

MSU Spring 2014

157 / 223

Interest Rate Swaps

We can interpret Rn in two ways. The first is to look at Rn as an average


forward rate:
Rn =

n
X

pj ij1,j

j=1

(1 + sj )j
pj = Pn
m
m=1 (1 + sm )
Pm
Note that 0 < pj < 1 and j=1 pj = 1 for positive spot rates.

Albert Cohen (MSU)

MATH 360: Theory of Investment and Credit

MSU Spring 2014

(132)

158 / 223

Interest Rate Swaps

The second way to interpret Rn is to look at Rn as an at par yield rate :


!
n 

X
(1 + sj )j
(1 + sj )j
Pn
Rn =

1
m
(1 + sj1 )j1
m=1 (1 + sm )
j=1
!
Pn
1
1
(133)
(1+s
j
j=1
(1+sj1 )j1
j)
1 (1 + sn )n
Pn
=
= Pn
m
m
m=1 (1 + sm )
m=1 (1 + sm )
= rn
as the numerator above is a telescoping series.

Albert Cohen (MSU)

MATH 360: Theory of Investment and Credit

MSU Spring 2014

159 / 223

Deferred and Non-Level Notional Interest Rate Swaps

Keeping with the analysis above, we have a Deferred Swap beginning in


k periods with value
(k)
Rn

n
X

(k)

pj

ij1,j

j=k
(k)
pj

Albert Cohen (MSU)

(134)

(1 + sj )j
= Pn
.
m
m=k (1 + sm )

MATH 360: Theory of Investment and Credit

MSU Spring 2014

160 / 223

Deferred and Non-Level Notional Interest Rate Swaps

Similarly, if the notional used to calculate the coupon varies with time, and
is defined as Fj at time j, then
0=

n
X
(Rn ik1,k )Fk
k=1

(135)

(1 + sk )k

then
n =
R

n
X

qj ij1,j

j=k

(136)

Fj (1 + sj )j
qj = Pn
.
m
m=k Fm (1 + sm )

Albert Cohen (MSU)

MATH 360: Theory of Investment and Credit

MSU Spring 2014

161 / 223

Example: Calculating the Swap Curve

Recall the table from Example 6.3:


Table: Example 6.3: Term Structure

Spot Rate
Forward Rate

Albert Cohen (MSU)

s1 = 0.05
i0,1 = 0.05

s2 = 0.1
i1,2 = 0.1524

MATH 360: Theory of Investment and Credit

s3 = 0.15
i2,3 = 0.2596

MSU Spring 2014

162 / 223

Example: Calculating the Swap Curve


We can now add to this table:
Table: Example 6.3: Term Structure

Spot
Fwd

s1 = 0.05
i0,1 = 0.05

Prob

p1 =

r3 =

1
1.05
1
1
+
+ 13
1.05
1.102
1.15

1
1.05
1
1.05

s2 = 0.1
i1,2 = 0.1524

0.05
+
1
1
+ 1.10
2 + 1.153

p2 =

1
1.102
1
1
+
+ 13
1.05
1.102
1.15

1
1.102
1
1.05

s3 = 0.15
i2,3 = 0.2596

0.1524
1
1.102

1
1.153

p3 =

1
1.153
1
1
+
+ 13
1.05
1.102
1.15

1
1.153
1
1.05

0.2596
1
1.102

1
1.153

= 0.1413.
(137)
HW: Compute r1 , r2 . Can you set up a spreadsheet to do this?
Albert Cohen (MSU)

MATH 360: Theory of Investment and Credit

MSU Spring 2014

163 / 223

Susceptibility: Single Factor Model


How does the PV of the cashflow change as the yield rate (term
structure) changes? (1st order changes)
Once quantified, this can be used as a measure of risk associated with
investing in this cashflow.

Albert Cohen (MSU)

MATH 360: Theory of Investment and Credit

MSU Spring 2014

164 / 223

Susceptibility: Single Factor Model


How does the PV of the cashflow change as the yield rate (term
structure) changes? (1st order changes)
Once quantified, this can be used as a measure of risk associated with
investing in this cashflow.
Consider the case where we have continuous compounding with a force of
interest . For a stream of payments, we have the present value, or up
front price, determined by P(). The only risk factor that could change
the value then is . If we were to equate this value with a zero-coupon
bond of unknown Duration D, then
P() = P(0)e D

Albert Cohen (MSU)

MATH 360: Theory of Investment and Credit

MSU Spring 2014

164 / 223

Susceptibility: Single Factor Model


How does the PV of the cashflow change as the yield rate (term
structure) changes? (1st order changes)
Once quantified, this can be used as a measure of risk associated with
investing in this cashflow.
Consider the case where we have continuous compounding with a force of
interest . For a stream of payments, we have the present value, or up
front price, determined by P(). The only risk factor that could change
the value then is . If we were to equate this value with a zero-coupon
bond of unknown Duration D, then
P() = P(0)e D
1 dP
D=
P d

Albert Cohen (MSU)

MATH 360: Theory of Investment and Credit

(138)

MSU Spring 2014

164 / 223

Susceptibility: Single Factor Model


How does the PV of the cashflow change as the yield rate (term
structure) changes? (1st order changes)
Once quantified, this can be used as a measure of risk associated with
investing in this cashflow.
Consider the case where we have continuous compounding with a force of
interest . For a stream of payments, we have the present value, or up
front price, determined by P(). The only risk factor that could change
the value then is . If we were to equate this value with a zero-coupon
bond of unknown Duration D, then
P() = P(0)e D
1 dP
D=
P d
This of course implies that
Albert Cohen (MSU)

dP
d

(138)

< 0.

MATH 360: Theory of Investment and Credit

MSU Spring 2014

164 / 223

Susceptibility and Sensitivity to Market Changes: Single


Factor Model
If the environment is one of annual compounding, then = ln (1 + i) and
we define
1 P(i) P(i + h)

h0 h
P(i)
d
= ln (P(i))
di
D := (1 + i) DM

DM := lim

(139)

For P(i) = (1 + i)n , we obtain


n
1+i
D=n

DM =

Albert Cohen (MSU)

MATH 360: Theory of Investment and Credit

(140)

MSU Spring 2014

165 / 223

Intuition

Increasing yield decreases present value. Thus, we take the negative


of derivative w.r.t yield.
The Modified Duration DM measures the ratio of rate of change of P
wrt i to the actual value P.
The Macaulay Duration D := (1 + i) DM says that the longer the
time maturity n, the more susceptible it is to yield change. For an
nyear zero coupon bond, D = n. Hence, the name duration
Can apply the linearization formula from calculus:
P(i + h) P(i) + P 0 (i) h = P(i) DM P(i) h
D
= P(i)
P(i) h
1+i

Albert Cohen (MSU)

MATH 360: Theory of Investment and Credit

MSU Spring 2014

(141)

166 / 223

Duration of General Cashflows

P=

n
X
m=1

Km
(1 + i)m

Pn
mKm
d
1 dP
m=1 (1+i)m+1
DM = ln (P(i)) =
= Pn
Km
di
P di
m=1 (1+i)m
D = (1 + i) DM =

n
X

(142)

m pm = E[m]

m=1

pm =

Albert Cohen (MSU)

Km
(1+i)m
Pn
Kl
l=1 (1+i)l

MATH 360: Theory of Investment and Credit

MSU Spring 2014

167 / 223

Duration of General Cashflows

Note also that

Km
n
m
dD
d X
(1+i)

=
m Pn
Kl
di
di
l
l=1
m=1

Albert Cohen (MSU)

(1+i)

MATH 360: Theory of Investment and Credit

MSU Spring 2014

168 / 223

Duration of General Cashflows

Note also that

Km
n
m
dD
d X
(1+i)

=
m Pn
Kl
di
di
l=1 (1+i)l
m=1

!2
n
n
X
1 X 2
m pm
m pm
=
1+i
m=1

Albert Cohen (MSU)

(143)

m=1

Var [m]
<0
1+i

MATH 360: Theory of Investment and Credit

MSU Spring 2014

168 / 223

Duration of a Perpetuity
Even though it has no finite term, we can still find the duration of a
perpetuity. In this case
1 dP
P di
 
1 d 1
= (1 + i) 1
di i
i
1
=1+
i

D = (1 + i)

Albert Cohen (MSU)

MATH 360: Theory of Investment and Credit

(144)

MSU Spring 2014

169 / 223

Duration of a Perpetuity
Even though it has no finite term, we can still find the duration of a
perpetuity. In this case
1 dP
P di
 
1 d 1
= (1 + i) 1
di i
i
1
=1+
i
An equivalent way to calculate this is to see that
P
k
(Ia) i
k=1 (1+i)k
D = P
=
1
a i
k=1 (1+i)k
D = (1 + i)

=
Albert Cohen (MSU)

1+i
i2
1
i

(144)

(145)

1
=1+
i

MATH 360: Theory of Investment and Credit

MSU Spring 2014

169 / 223

Example 7.3 Duration of a Coupon Bond

In the case of a coupon bond, we have K1 = K2 = .. = Kn1 = F r , and


Kn = F + F r , and so
Pn
nF
mF r
m=1 (1+i)m + (1+i)n
D = Pn
F r
F
m=1 (1+i)m + (1+i)n


n
(1+i)an i (1+i)
n
n
(146)
r
+ (1+i)
n
i
=
1
r an i + (1+i)
n

Albert Cohen (MSU)

MATH 360: Theory of Investment and Credit

MSU Spring 2014

170 / 223

Bonds and Macaulay Duration: Example from previous


SOA Exam FM

A bond with Face F will pay a coupon of F r at the end of each of the
next three years. It wil also pay the face value of F at the end of the
three-year period. The bonds duration (Macaulay duration) when valued
using an annual effective interest rate of 20% is X. Calculate X .

Albert Cohen (MSU)

MATH 360: Theory of Investment and Credit

MSU Spring 2014

171 / 223

Bonds and Macaulay Duration: Example from previous


SOA Exam FM
Recall that the Macaulay Duration over n time periods is defined as an
expection of sorts:
Pn
ti PV (i)
D = Pi=1
(147)
n
j=1 PV (j)
where the PV (i) are the present values of the revenue received at time ti .
For our present case, we have
1
D=

Albert Cohen (MSU)

r
1.2

F (1+r )
(1.2)3
F (1+r )
F r
+ (1.2)3
(1.2)2
(1+r )
r
(1.2)
2 + 3 (1.2)3
)
r
+ (1+r
(1.2)2
(1.2)3

+2

F r
1.2

1
=

F r
1.2

+2

r
1.2

F r
(1.2)2

+3

MATH 360: Theory of Investment and Credit

(148)

MSU Spring 2014

172 / 223

Portfolio Duration
ImagineP
that P(i) is the price of a portfolio of income streams:
P(i) = nk=1 Pk (i). Then
Pn
P 0 (i)
P 0 (i)
= (1 + i) k=1 k
D = (1 + i)
P(i)
P(i)
n
n
0
X P (i)
X
Pk (i) Pk0 (i)
k
= (1 + i)
= (1 + i)

P(i)
P(i) Pk (i)
k=1

n
X

k=1

(149)

Dk qk

k=1



Pk0 (i) Pk (i)
(Dk , qk ) = (1 + i)
,
Pk (i) P(i)
and so the portfolio duration is the weighted average of the individual
durations.
Albert Cohen (MSU)

MATH 360: Theory of Investment and Credit

MSU Spring 2014

173 / 223

Portfolio Duration

Smith holds a portfolio of a 20 year 100 bond bought at par and with
duration of 8, a stock purchased for 50 with duration 7 and an 30 year
annuity purchased for 50 with duration 5. What is the portfolio duration?

Albert Cohen (MSU)

MATH 360: Theory of Investment and Credit

MSU Spring 2014

174 / 223

Portfolio Duration

Smith holds a portfolio of a 20 year 100 bond bought at par and with
duration of 8, a stock purchased for 50 with duration 7 and an 30 year
annuity purchased for 50 with duration 5. What is the portfolio duration?
D =8

Albert Cohen (MSU)

100
50
50
+7
+5
=7
200
200
200

MATH 360: Theory of Investment and Credit

(150)

MSU Spring 2014

174 / 223

Cashflow Immunization and Convexity

An institution such as a bank or insurance company will have both assets


and liabilities which may come due at different times. These may be
structured so that the net present value of the institution is zero. Hence,
for a given i,
P(i) 0

(151)

If the rate i is perturbed slightly, the institution may find itself with a
negative NPV. To counter this risk, the institution may structure its assets
and liabilities such that P is a local minimum at i.

Albert Cohen (MSU)

MATH 360: Theory of Investment and Credit

MSU Spring 2014

175 / 223

Cashflow Immunization and Convexity

To explain this further, define the Convexity


:=

P 00 (i)
P(i)

(152)

and so
P(i + h) P(i)
DM h + h2
P(i)

(153)

If P 0 (i) = 0 and (i) > 0, then DM = 0 and we have a local minimum

Albert Cohen (MSU)

MATH 360: Theory of Investment and Credit

MSU Spring 2014

176 / 223

Cashflow Immunization Example

Suppose a bank has promised its investors a one-year 5% return on


deposits of 100. The bank has 50 per investor on hand, is able to purchase
perpetuities that pay one per year and one year zero-coupon bonds that
yield 10%. How should the bank structure its assets so they are
immunized from interest rate risk?

Albert Cohen (MSU)

MATH 360: Theory of Investment and Credit

MSU Spring 2014

177 / 223

Cashflow Immunization Example


At rate i, we have the NPV of x units perpetuity and y units of bond
minus the NPV of the liability of the one-year withdrawal as
1.1 y
105
x
+

i
1+i
1+i
x
1.1 y 105
0
P (i) = 2
i
(1 + i)2
1.1 y 105
x
P 00 (i) = 2 3 + 2
i
(1 + i)3
P(i) = 50 +

(154)

For whatever rate i we fix, if we set


x = 50i 2
y=

then P(i) = P 0 (i) = 0 and P 00 (i) =


minimum.
Albert Cohen (MSU)

(155)

105 50 (1 + i)2
1.1
100
i(1+i)

> 0 so we have a local

MATH 360: Theory of Investment and Credit

MSU Spring 2014

178 / 223

Some Comments

Book, or Amortized, value is calculated using fixed rate


Market value determined via market term structure
The spot rate used in this term structure varies with time
This leads to difference between Book and Market value of cashflow
These values converge as time approaches maturity; investor can hold
on to his investment to realize original yield rate
Before maturity, however, the cashflow is valued at market rate
Previous analysis is for flat term structure. To generalize to normal or
inverted term structure, require new definitions and multivariable
calculus

Albert Cohen (MSU)

MATH 360: Theory of Investment and Credit

MSU Spring 2014

179 / 223

Basic Ideas and Beyond


As we have seen, interest rate fluctuation can lead to possible
shortfalls that banks may wish to immunize their portfolio against.
Currency fluctuation also leads to operational risk, and should
factor into planning. For example, should a band tour Europe or
America this summer ? Click here for an insightful article by Neil
Shah in the Wall Street Journal TM , with comment from the
manager of a very prominent rock band
Interest rates are only one risk factor. Another very real factor is
known as longevity risk, which is due to the possibility that a
pensioner may live longer than expected. Hedging against such a
possibility is extremely important, and a topic we hope to cover in
STT 455 456. In the meantime, please consult the paper by Tsai,
Tzeng, and Wang on Hedging Longevity Risk When Interest Rates
Are Uncertain
Albert Cohen (MSU)

MATH 360: Theory of Investment and Credit

MSU Spring 2014

180 / 223

Probability Space
Let us define an event as a point in the set of all possible outcomes .
This includes the events The stock doubled in price over two trading
periods or the average stock price over ten years was 10 dollars.

Albert Cohen (MSU)

MATH 360: Theory of Investment and Credit

MSU Spring 2014

181 / 223

Probability Space
Let us define an event as a point in the set of all possible outcomes .
This includes the events The stock doubled in price over two trading
periods or the average stock price over ten years was 10 dollars.
In our initial case, we will consider the simple binary space
= {H, T } for a one-period asset evolution. So, given an initial
value S0 , we have the final value S1 (), with

Albert Cohen (MSU)

MATH 360: Theory of Investment and Credit

MSU Spring 2014

181 / 223

Probability Space
Let us define an event as a point in the set of all possible outcomes .
This includes the events The stock doubled in price over two trading
periods or the average stock price over ten years was 10 dollars.
In our initial case, we will consider the simple binary space
= {H, T } for a one-period asset evolution. So, given an initial
value S0 , we have the final value S1 (), with
S1 (H) = uS0 , S1 (T ) = dS0

Albert Cohen (MSU)

MATH 360: Theory of Investment and Credit

(156)

MSU Spring 2014

181 / 223

Probability Space
Let us define an event as a point in the set of all possible outcomes .
This includes the events The stock doubled in price over two trading
periods or the average stock price over ten years was 10 dollars.
In our initial case, we will consider the simple binary space
= {H, T } for a one-period asset evolution. So, given an initial
value S0 , we have the final value S1 (), with
S1 (H) = uS0 , S1 (T ) = dS0

(156)

with d < 1 < u. Hence, a stock increases or decreases in price,


according to the flip of a coin.

Albert Cohen (MSU)

MATH 360: Theory of Investment and Credit

MSU Spring 2014

181 / 223

Probability Space
Let us define an event as a point in the set of all possible outcomes .
This includes the events The stock doubled in price over two trading
periods or the average stock price over ten years was 10 dollars.
In our initial case, we will consider the simple binary space
= {H, T } for a one-period asset evolution. So, given an initial
value S0 , we have the final value S1 (), with
S1 (H) = uS0 , S1 (T ) = dS0

(156)

with d < 1 < u. Hence, a stock increases or decreases in price,


according to the flip of a coin.
Let P be the probability measure associated with these events:
P[H] = p = 1 P[T ]

Albert Cohen (MSU)

MATH 360: Theory of Investment and Credit

(157)

MSU Spring 2014

181 / 223

Arbitrage

Assume that S0 (1 + r ) > uS0

Albert Cohen (MSU)

MATH 360: Theory of Investment and Credit

MSU Spring 2014

182 / 223

Arbitrage

Assume that S0 (1 + r ) > uS0


Where is the risk involved with investing in the asset S ?

Albert Cohen (MSU)

MATH 360: Theory of Investment and Credit

MSU Spring 2014

182 / 223

Arbitrage

Assume that S0 (1 + r ) > uS0


Where is the risk involved with investing in the asset S ?
Assume that S0 (1 + r ) < dS0

Albert Cohen (MSU)

MATH 360: Theory of Investment and Credit

MSU Spring 2014

182 / 223

Arbitrage

Assume that S0 (1 + r ) > uS0


Where is the risk involved with investing in the asset S ?
Assume that S0 (1 + r ) < dS0
Why would anyone hold a bank account (zero-coupon bond)?

Albert Cohen (MSU)

MATH 360: Theory of Investment and Credit

MSU Spring 2014

182 / 223

Arbitrage

Assume that S0 (1 + r ) > uS0


Where is the risk involved with investing in the asset S ?
Assume that S0 (1 + r ) < dS0
Why would anyone hold a bank account (zero-coupon bond)?
Lemma Arbitrage free d < 1 + r < u

Albert Cohen (MSU)

MATH 360: Theory of Investment and Credit

MSU Spring 2014

182 / 223

Derivative Pricing
Let S1 () be the price of an underlying asset at time 1. Define the
following instruments:

Albert Cohen (MSU)

MATH 360: Theory of Investment and Credit

MSU Spring 2014

183 / 223

Derivative Pricing
Let S1 () be the price of an underlying asset at time 1. Define the
following instruments:
1
B
1+r , V1 () = 1
0, V1F = S1 () F

Zero-Coupon Bond : V0B =


Forward Contract : V0F =

Call Option : V1C () = max(S1 () K , 0)


Put Option : V1P () = max(K S1 (), 0)

Albert Cohen (MSU)

MATH 360: Theory of Investment and Credit

MSU Spring 2014

183 / 223

Derivative Pricing
Let S1 () be the price of an underlying asset at time 1. Define the
following instruments:
1
B
1+r , V1 () = 1
0, V1F = S1 () F

Zero-Coupon Bond : V0B =


Forward Contract : V0F =

Call Option : V1C () = max(S1 () K , 0)


Put Option : V1P () = max(K S1 (), 0)
In both the Call and Put option, K is known as the Strike.

Albert Cohen (MSU)

MATH 360: Theory of Investment and Credit

MSU Spring 2014

183 / 223

Derivative Pricing
Let S1 () be the price of an underlying asset at time 1. Define the
following instruments:
1
B
1+r , V1 () = 1
0, V1F = S1 () F

Zero-Coupon Bond : V0B =


Forward Contract : V0F =

Call Option : V1C () = max(S1 () K , 0)


Put Option : V1P () = max(K S1 (), 0)
In both the Call and Put option, K is known as the Strike.
Once again, a Forward Contract is a deal that is locked in at time 0 for
initial price 0, but requires at time 1 the buyer to purchase the asset for
price F .

Albert Cohen (MSU)

MATH 360: Theory of Investment and Credit

MSU Spring 2014

183 / 223

Derivative Pricing
Let S1 () be the price of an underlying asset at time 1. Define the
following instruments:
1
B
1+r , V1 () = 1
0, V1F = S1 () F

Zero-Coupon Bond : V0B =


Forward Contract : V0F =

Call Option : V1C () = max(S1 () K , 0)


Put Option : V1P () = max(K S1 (), 0)
In both the Call and Put option, K is known as the Strike.
Once again, a Forward Contract is a deal that is locked in at time 0 for
initial price 0, but requires at time 1 the buyer to purchase the asset for
price F .
What is the value V0 of the above put and call options?

Albert Cohen (MSU)

MATH 360: Theory of Investment and Credit

MSU Spring 2014

183 / 223

Put-Call Parity
Can we replicate a forward contract using zero coupon bonds and put and
call options?

Albert Cohen (MSU)

MATH 360: Theory of Investment and Credit

MSU Spring 2014

184 / 223

Put-Call Parity
Can we replicate a forward contract using zero coupon bonds and put and
call options?
Yes: The final value of a replicating strategy X has value

Albert Cohen (MSU)

MATH 360: Theory of Investment and Credit

MSU Spring 2014

184 / 223

Put-Call Parity
Can we replicate a forward contract using zero coupon bonds and put and
call options?
Yes: The final value of a replicating strategy X has value
V1C V1P + (K F ) = S1 F = X1 ()

Albert Cohen (MSU)

MATH 360: Theory of Investment and Credit

MSU Spring 2014

(158)

184 / 223

Put-Call Parity
Can we replicate a forward contract using zero coupon bonds and put and
call options?
Yes: The final value of a replicating strategy X has value
V1C V1P + (K F ) = S1 F = X1 ()

(158)

This is achieved (replicated) by


Purchasing one call option
Selling one put option
Purchasing K F zero coupon bonds
all at time 0.

Albert Cohen (MSU)

MATH 360: Theory of Investment and Credit

MSU Spring 2014

184 / 223

Put-Call Parity
Can we replicate a forward contract using zero coupon bonds and put and
call options?
Yes: The final value of a replicating strategy X has value
V1C V1P + (K F ) = S1 F = X1 ()

(158)

This is achieved (replicated) by


Purchasing one call option
Selling one put option
Purchasing K F zero coupon bonds
all at time 0.
Since this strategy must have zero initial value, we obtain
V0C V0P =
Albert Cohen (MSU)

F K
1+r

MATH 360: Theory of Investment and Credit

(159)

MSU Spring 2014

184 / 223

Put-Call Parity
Can we replicate a forward contract using zero coupon bonds and put and
call options?
Yes: The final value of a replicating strategy X has value
V1C V1P + (K F ) = S1 F = X1 ()

(158)

This is achieved (replicated) by


Purchasing one call option
Selling one put option
Purchasing K F zero coupon bonds
all at time 0.
Since this strategy must have zero initial value, we obtain
F K
1+r
Question: How would this change in a multi-period model?
V0C V0P =

Albert Cohen (MSU)

MATH 360: Theory of Investment and Credit

MSU Spring 2014

(159)

184 / 223

General Derivative Pricing -One period model

If we begin with some initial capital X0 , then we end with X1 (). To price
a derivative, we need to match
X1 () = V1 ()

(160)

to have X0 = V0 , the price of the derivative we seek.


A strategy by the pair (X0 , 0 ) wherein
X0 is the initial capital
0 is the initial number of shares (units of underlying asset.)
What does the sign of 0 indicate?

Albert Cohen (MSU)

MATH 360: Theory of Investment and Credit

MSU Spring 2014

185 / 223

Replicating Strategy

Albert Cohen (MSU)

MATH 360: Theory of Investment and Credit

MSU Spring 2014

186 / 223

Replicating Strategy
Initial holding in bond (bank account) is X0 0 S0
Value of portfolio at maturity is
X1 () = (X0 0 S0 )(1 + r ) + 0 S1 ()

Albert Cohen (MSU)

MATH 360: Theory of Investment and Credit

MSU Spring 2014

(161)

186 / 223

Replicating Strategy
Initial holding in bond (bank account) is X0 0 S0
Value of portfolio at maturity is
X1 () = (X0 0 S0 )(1 + r ) + 0 S1 ()

(161)

Pathwise, we compute

V1 (H) = (X0 0 S0 )(1 + r ) + 0 uS0


V1 (T ) = (X0 0 S0 )(1 + r ) + 0 dS0

Albert Cohen (MSU)

MATH 360: Theory of Investment and Credit

MSU Spring 2014

186 / 223

Replicating Strategy
Initial holding in bond (bank account) is X0 0 S0
Value of portfolio at maturity is
X1 () = (X0 0 S0 )(1 + r ) + 0 S1 ()

(161)

Pathwise, we compute

V1 (H) = (X0 0 S0 )(1 + r ) + 0 uS0


V1 (T ) = (X0 0 S0 )(1 + r ) + 0 dS0

Algebra yields
0 =
Albert Cohen (MSU)

V1 (H) V1 (T )
(u d)S0

MATH 360: Theory of Investment and Credit

(162)
MSU Spring 2014

186 / 223

Risk Neutral Probability

Let us assume the existence of a pair (


p , q) of positive numbers, and use
these to multiply our pricing equation(s):

pV1 (H) = p(X0 0 S0 )(1 + r ) + p0 uS0


qV1 (T ) = q(X0 0 S0 )(1 + r ) + q0 dS0

Albert Cohen (MSU)

MATH 360: Theory of Investment and Credit

MSU Spring 2014

187 / 223

Risk Neutral Probability

Let us assume the existence of a pair (


p , q) of positive numbers, and use
these to multiply our pricing equation(s):

pV1 (H) = p(X0 0 S0 )(1 + r ) + p0 uS0


qV1 (T ) = q(X0 0 S0 )(1 + r ) + q0 dS0

Addition yields

X0 (1 + r ) + 0 S0 (
p u + qd (1 + r )) = pV1 (H) + qV1 (T )

Albert Cohen (MSU)

MATH 360: Theory of Investment and Credit

MSU Spring 2014

(163)

187 / 223

If we constrain
0 = pu + qd (1 + r )
1 = p + q
0 p
0 q

Albert Cohen (MSU)

MATH 360: Theory of Investment and Credit

MSU Spring 2014

188 / 223

If we constrain
0 = pu + qd (1 + r )
1 = p + q
0 p
0 q
where
then we have a risk neutral probability P
V0 = X0 =

1
pV1 (H) + qV1 (T )
E[V1 ] =
1+r
1+r

(164)

with
1+r d
ud
u

(1 + r )
MATH 360: Theory of Investment and Credit
p =

Albert Cohen (MSU)

MSU Spring 2014

188 / 223

Example: Pricing a forward contract

Consider the case of a stock with


S0 = 100
u = 1.2
d = 0.8
r = 0.05
Then the forward price is computed via

Albert Cohen (MSU)

MATH 360: Theory of Investment and Credit

MSU Spring 2014

189 / 223

Example: Pricing a forward contract

Consider the case of a stock with


S0 = 100
u = 1.2
d = 0.8
r = 0.05
Then the forward price is computed via
0=

Albert Cohen (MSU)

1
1]
E[S1 F ] F = E[S
1+r

MATH 360: Theory of Investment and Credit

(165)

MSU Spring 2014

189 / 223

This leads to the explicit price

F = puS0 + qdS0
= (0.625)(1.2)(100) + (0.375)(0.8)(100) = 105

Albert Cohen (MSU)

MATH 360: Theory of Investment and Credit

MSU Spring 2014

190 / 223

This leads to the explicit price

F = puS0 + qdS0
= (0.625)(1.2)(100) + (0.375)(0.8)(100) = 105

Homework Question: What is the price of a call option in the case


above,with strike K = 95?

Albert Cohen (MSU)

MATH 360: Theory of Investment and Credit

MSU Spring 2014

190 / 223

General one period risk neutral measure

We define a finite set of outcomes {1 , 2 , ..., n } and any


subcollection of outcomes A an event.

Albert Cohen (MSU)

MATH 360: Theory of Investment and Credit

MSU Spring 2014

191 / 223

General one period risk neutral measure

We define a finite set of outcomes {1 , 2 , ..., n } and any


subcollection of outcomes A an event.
not necessarily the
Furthermore, we define a probability measure P,
physical measure P to be risk neutral if

P[]
>0
1]
X0 = 1 E[X
1+r

for all strategies X .

Albert Cohen (MSU)

MATH 360: Theory of Investment and Credit

MSU Spring 2014

191 / 223

General one period risk neutral measure

The measure is indifferent to investing in a zero-coupon bond, or a


risky asset X

Albert Cohen (MSU)

MATH 360: Theory of Investment and Credit

MSU Spring 2014

192 / 223

General one period risk neutral measure

The measure is indifferent to investing in a zero-coupon bond, or a


risky asset X
The same initial capital X0 in both cases produces the same
average return after one period.

Albert Cohen (MSU)

MATH 360: Theory of Investment and Credit

MSU Spring 2014

192 / 223

General one period risk neutral measure

The measure is indifferent to investing in a zero-coupon bond, or a


risky asset X
The same initial capital X0 in both cases produces the same
average return after one period.
Not the physical measure attached by observation, experts, etc..

Albert Cohen (MSU)

MATH 360: Theory of Investment and Credit

MSU Spring 2014

192 / 223

General one period risk neutral measure

The measure is indifferent to investing in a zero-coupon bond, or a


risky asset X
The same initial capital X0 in both cases produces the same
average return after one period.
Not the physical measure attached by observation, experts, etc..
In fact, physical measure has no impact on pricing

Albert Cohen (MSU)

MATH 360: Theory of Investment and Credit

MSU Spring 2014

192 / 223

Example: Risk Neutral measure for trinomial case


Assume that = {1 , 2 , 3 } with
S1 (1 ) = uS0
S1 (2 ) = S0
S1 (3 ) = dS0

Albert Cohen (MSU)

MATH 360: Theory of Investment and Credit

MSU Spring 2014

193 / 223

Example: Risk Neutral measure for trinomial case


Assume that = {1 , 2 , 3 } with
S1 (1 ) = uS0
S1 (2 ) = S0
S1 (3 ) = dS0
Given a payoff V1 () to replicate, are we assured that a replicating
strategy exists?

Albert Cohen (MSU)

MATH 360: Theory of Investment and Credit

MSU Spring 2014

193 / 223

Example: Risk Neutral measure for trinomial case


Assume that = {1 , 2 , 3 } with
S1 (1 ) = uS0
S1 (2 ) = S0
S1 (3 ) = dS0
Given a payoff V1 () to replicate, are we assured that a replicating
strategy exists?
Homework: Try our first example with
S0 = 100
r = 0.05, u = 1.2, d = 0.8
V1 () = (S1 () > 90)
Albert Cohen (MSU)

MATH 360: Theory of Investment and Credit

MSU Spring 2014

193 / 223

Existence of Risk Neutral measure

be a probability measure on a finite space . The following are


Let P
equivalent:

Albert Cohen (MSU)

MATH 360: Theory of Investment and Credit

MSU Spring 2014

194 / 223

Existence of Risk Neutral measure

be a probability measure on a finite space . The following are


Let P
equivalent:
is a risk neutral measure
P

Albert Cohen (MSU)

MATH 360: Theory of Investment and Credit

MSU Spring 2014

194 / 223

Existence of Risk Neutral measure

be a probability measure on a finite space . The following are


Let P
equivalent:
is a risk neutral measure
P
For all traded securities S i , S0i =

Albert Cohen (MSU)

1
1+r E

 i
S1

MATH 360: Theory of Investment and Credit

MSU Spring 2014

194 / 223

Existence of Risk Neutral measure

be a probability measure on a finite space . The following are


Let P
equivalent:
is a risk neutral measure
P
For all traded securities S i , S0i =

1
1+r E

 i
S1

Proof: Homework (Hint: One direction is much easier than others. Also,
strategies are linear in the underlying asset.)

Albert Cohen (MSU)

MATH 360: Theory of Investment and Credit

MSU Spring 2014

194 / 223

Complete Markets

A market is complete if it is arbitrage free and every non-traded asset can


be replicated.

Albert Cohen (MSU)

MATH 360: Theory of Investment and Credit

MSU Spring 2014

195 / 223

Complete Markets

A market is complete if it is arbitrage free and every non-traded asset can


be replicated.
Fundamental Theorem of Asset Pricing 1: A market is arbitrage free
iff there exists a risk neutral measure
Fundamental Theorem of Asset Pricing 2: A market is complete iff
there exists exactly one risk neutral measure

Albert Cohen (MSU)

MATH 360: Theory of Investment and Credit

MSU Spring 2014

195 / 223

2-period pricing
Consider the case

r = 0.05, S0 = 100
S1 (H) = 1.2S0
S1 (T ) = 0.8S0
S2 (HH) = 1.2S1 (H)
S2 (HT ) = 0.8S1 (H)
S2 (TH) = 1.2S1 (T )
S2 (TT ) = 0.8S1 (T )

Albert Cohen (MSU)

MATH 360: Theory of Investment and Credit

MSU Spring 2014

196 / 223

2-period pricing
Consider the case

r = 0.05, S0 = 100
S1 (H) = 1.2S0
S1 (T ) = 0.8S0
S2 (HH) = 1.2S1 (H)
S2 (HT ) = 0.8S1 (H)
S2 (TH) = 1.2S1 (T )
S2 (TT ) = 0.8S1 (T )

Now price a digital option that has payoff V2 := (S2 100)


Albert Cohen (MSU)

MATH 360: Theory of Investment and Credit

MSU Spring 2014

196 / 223

We can do this for 2-period problems

Albert Cohen (MSU)

MATH 360: Theory of Investment and Credit

MSU Spring 2014

197 / 223

We can do this for 2-period problems


Case by Case, or..
by developing a general theory for multi-period asset pricing

Albert Cohen (MSU)

MATH 360: Theory of Investment and Credit

MSU Spring 2014

197 / 223

We can do this for 2-period problems


Case by Case, or..
by developing a general theory for multi-period asset pricing
In the latter method, we need a general framework to carry out our
computations

Albert Cohen (MSU)

MATH 360: Theory of Investment and Credit

MSU Spring 2014

197 / 223

Multi-Period Pricing - Introduction

Again, we define the finite set of outcomes and any subcollection of


outcomes A an event.

Albert Cohen (MSU)

MATH 360: Theory of Investment and Credit

MSU Spring 2014

198 / 223

Multi-Period Pricing - Introduction

Again, we define the finite set of outcomes and any subcollection of


outcomes A an event.
How does this relate to the case of 2 consecutive coin flips

Albert Cohen (MSU)

MATH 360: Theory of Investment and Credit

MSU Spring 2014

198 / 223

Multi-Period Pricing - Introduction

Again, we define the finite set of outcomes and any subcollection of


outcomes A an event.
How does this relate to the case of 2 consecutive coin flips
{HH, HT , TH, TT }

Albert Cohen (MSU)

MATH 360: Theory of Investment and Credit

MSU Spring 2014

198 / 223

Multi-Period Pricing - Introduction

Again, we define the finite set of outcomes and any subcollection of


outcomes A an event.
How does this relate to the case of 2 consecutive coin flips
{HH, HT , TH, TT }
Set of events includes statements like at least one head

Albert Cohen (MSU)

MATH 360: Theory of Investment and Credit

MSU Spring 2014

198 / 223

Multi-Period Pricing - Introduction

Again, we define the finite set of outcomes and any subcollection of


outcomes A an event.
How does this relate to the case of 2 consecutive coin flips
{HH, HT , TH, TT }
Set of events includes statements like at least one head
= {HH, HT , TH}

Albert Cohen (MSU)

MATH 360: Theory of Investment and Credit

MSU Spring 2014

198 / 223

algebras

Given a non-empty set of outcomes, a algebra F is a collection of


subsets of that satisfies

Albert Cohen (MSU)

MATH 360: Theory of Investment and Credit

MSU Spring 2014

199 / 223

algebras

Given a non-empty set of outcomes, a algebra F is a collection of


subsets of that satisfies
F

Albert Cohen (MSU)

MATH 360: Theory of Investment and Credit

MSU Spring 2014

199 / 223

algebras

Given a non-empty set of outcomes, a algebra F is a collection of


subsets of that satisfies
F
A F Ac F

Albert Cohen (MSU)

MATH 360: Theory of Investment and Credit

MSU Spring 2014

199 / 223

algebras

Given a non-empty set of outcomes, a algebra F is a collection of


subsets of that satisfies
F
A F Ac F
A1 , A2 , A3 , .... F
n=1 An F

Albert Cohen (MSU)

MATH 360: Theory of Investment and Credit

MSU Spring 2014

199 / 223

algebras

Given a non-empty set of outcomes, a algebra F is a collection of


subsets of that satisfies
F
A F Ac F
A1 , A2 , A3 , .... F
n=1 An F
Some Examples

Albert Cohen (MSU)

MATH 360: Theory of Investment and Credit

MSU Spring 2014

199 / 223

algebras

Given a non-empty set of outcomes, a algebra F is a collection of


subsets of that satisfies
F
A F Ac F
A1 , A2 , A3 , .... F
n=1 An F
Some Examples
F0 = {, }

Albert Cohen (MSU)

MATH 360: Theory of Investment and Credit

MSU Spring 2014

199 / 223

algebras

Given a non-empty set of outcomes, a algebra F is a collection of


subsets of that satisfies
F
A F Ac F
A1 , A2 , A3 , .... F
n=1 An F
Some Examples
F0 = {, }
F1 = {, , {HH, HT }, {TT , TH}}

Albert Cohen (MSU)

MATH 360: Theory of Investment and Credit

MSU Spring 2014

199 / 223

algebras

Given a non-empty set of outcomes, a algebra F is a collection of


subsets of that satisfies
F
A F Ac F
A1 , A2 , A3 , .... F
n=1 An F
Some Examples
F0 = {, }
F1 = {, , {HH, HT }, {TT , TH}}
F2 = {, , {HH}, {HT }, {TT }, {TH}, ....}

Albert Cohen (MSU)

MATH 360: Theory of Investment and Credit

MSU Spring 2014

199 / 223

algebras

Given a non-empty set of outcomes, a algebra F is a collection of


subsets of that satisfies
F
A F Ac F
A1 , A2 , A3 , .... F
n=1 An F
Some Examples
F0 = {, }
F1 = {, , {HH, HT }, {TT , TH}}
F2 = {, , {HH}, {HT }, {TT }, {TH}, ....}
F2 is completed by taking all unions of , , {HH}, {HT }, {TT }, {TH}.

Albert Cohen (MSU)

MATH 360: Theory of Investment and Credit

MSU Spring 2014

199 / 223

Notice that F0 F1 F2 .
Correspondingly, given an , we define a

Albert Cohen (MSU)

MATH 360: Theory of Investment and Credit

MSU Spring 2014

200 / 223

Notice that F0 F1 F2 .
Correspondingly, given an , we define a
Filtration as ..

Albert Cohen (MSU)

MATH 360: Theory of Investment and Credit

MSU Spring 2014

200 / 223

Notice that F0 F1 F2 .
Correspondingly, given an , we define a
Filtration as ..
a sequence of algebras F0 , F1 , F2 , ..., Fn , ... such that

Albert Cohen (MSU)

MATH 360: Theory of Investment and Credit

MSU Spring 2014

200 / 223

Notice that F0 F1 F2 .
Correspondingly, given an , we define a
Filtration as ..
a sequence of algebras F0 , F1 , F2 , ..., Fn , ... such that
F0 F1 F2 ... Fn ...

Albert Cohen (MSU)

MATH 360: Theory of Investment and Credit

MSU Spring 2014

200 / 223

Notice that F0 F1 F2 .
Correspondingly, given an , we define a
Filtration as ..
a sequence of algebras F0 , F1 , F2 , ..., Fn , ... such that
F0 F1 F2 ... Fn ...
and F = () as the algebra of all subsets of .

Albert Cohen (MSU)

MATH 360: Theory of Investment and Credit

MSU Spring 2014

200 / 223

Notice that F0 F1 F2 .
Correspondingly, given an , we define a
Filtration as ..
a sequence of algebras F0 , F1 , F2 , ..., Fn , ... such that
F0 F1 F2 ... Fn ...
and F = () as the algebra of all subsets of .
Given a pair (, F), we define a Random Variable X () as a mapping
X :R

Albert Cohen (MSU)

MATH 360: Theory of Investment and Credit

MSU Spring 2014

200 / 223

Given a pair (, F) and random variable X ,


(X ) = the collection of all sets { | X () A R}

Albert Cohen (MSU)

MATH 360: Theory of Investment and Credit

MSU Spring 2014

201 / 223

Given a pair (, F) and random variable X ,


(X ) = the collection of all sets { | X () A R}
Given a G F, we say that X is G measurable if

Albert Cohen (MSU)

MATH 360: Theory of Investment and Credit

MSU Spring 2014

201 / 223

Given a pair (, F) and random variable X ,


(X ) = the collection of all sets { | X () A R}
Given a G F, we say that X is G measurable if
A (X ) A G

Albert Cohen (MSU)

MATH 360: Theory of Investment and Credit

MSU Spring 2014

201 / 223

Given a pair (, F) and random variable X ,


(X ) = the collection of all sets { | X () A R}
Given a G F, we say that X is G measurable if
A (X ) A G

A Probability Space for us will be the triple (, F, P), where

Albert Cohen (MSU)

MATH 360: Theory of Investment and Credit

MSU Spring 2014

201 / 223

P : F [0, 1]
P[] = 0
For any countable disjoint sets A1 , A2 , ... F
P [
n=1 An ] =

Albert Cohen (MSU)

n=1

P[An ]

MATH 360: Theory of Investment and Credit

MSU Spring 2014

202 / 223

And so
P[A] :=

P[]
PA
P
E[X ] := X ()P[] = nk=1 xk P[{X () = xk }]
h
i
with Variance := E (X E[X ])2

Albert Cohen (MSU)

MATH 360: Theory of Investment and Credit

MSU Spring 2014

203 / 223

Conditional Expectation
Let us return to the two flip model.

Albert Cohen (MSU)

MATH 360: Theory of Investment and Credit

MSU Spring 2014

204 / 223

Conditional Expectation
Let us return to the two flip model.
and know the value S1 , can we
If we are give a probability measure P,
estimate the value S2 given (conditional on) this information?

Albert Cohen (MSU)

MATH 360: Theory of Investment and Credit

MSU Spring 2014

204 / 223

Conditional Expectation
Let us return to the two flip model.
and know the value S1 , can we
If we are give a probability measure P,
estimate the value S2 given (conditional on) this information?
An Example:

Albert Cohen (MSU)

MATH 360: Theory of Investment and Credit

MSU Spring 2014

204 / 223

Conditional Expectation
Let us return to the two flip model.
and know the value S1 , can we
If we are give a probability measure P,
estimate the value S2 given (conditional on) this information?
An Example:
S0 = 100, S1 (H) = 120, S1 (T ) = 80
p = 0.4, q = 0.6 for each flip
= (1 , 2 ) - each flip is independent, and a path is the total path
generated by both flips

Albert Cohen (MSU)

MATH 360: Theory of Investment and Credit

MSU Spring 2014

204 / 223

Conditional Expectation
Let us return to the two flip model.
and know the value S1 , can we
If we are give a probability measure P,
estimate the value S2 given (conditional on) this information?
An Example:
S0 = 100, S1 (H) = 120, S1 (T ) = 80
p = 0.4, q = 0.6 for each flip
= (1 , 2 ) - each flip is independent, and a path is the total path
generated by both flips
Given this set-up, compute

Albert Cohen (MSU)

MATH 360: Theory of Investment and Credit

MSU Spring 2014

204 / 223

Conditional Expectation
Let us return to the two flip model.
and know the value S1 , can we
If we are give a probability measure P,
estimate the value S2 given (conditional on) this information?
An Example:
S0 = 100, S1 (H) = 120, S1 (T ) = 80
p = 0.4, q = 0.6 for each flip
= (1 , 2 ) - each flip is independent, and a path is the total path
generated by both flips
Given this set-up, compute
[S2 | S1 ] (1 )
E

Albert Cohen (MSU)

MATH 360: Theory of Investment and Credit

(166)

MSU Spring 2014

204 / 223

This means if we take X = S1 , for any A (S1 )


X
A

S2 ()P[]
=

[S2 | S1 ] ()P[]

(167)

Can extend this to any sub algebra of (S2 )

Albert Cohen (MSU)

MATH 360: Theory of Investment and Credit

MSU Spring 2014

205 / 223

Assume X F and G F
Tower Property: If G1 G2 , then
h
i
[X | G1 ] = E
E
[X | G2 ] | G1
E

(168)

[X | G ] = X
If X is G measurable, then E

Albert Cohen (MSU)

MATH 360: Theory of Investment and Credit

MSU Spring 2014

206 / 223

[| X |] < , then
Jensens Inequality: If f : R R is convex, and E


[f (X ) | G ] f E
[X | G ]
E
(169)

[X | G ] 0
If P[{X
0}] = 1, then E
Linearity
Independence: If X does not depend on the information contained in
G , then
E [X | G ] = E [X ]
(170)

Albert Cohen (MSU)

MATH 360: Theory of Investment and Credit

MSU Spring 2014

207 / 223

Taking out what is known: If X is dependent only on the information


contained in G , then
[XY | G ] = X E
[Y | G ]
E

(171)

Example: If G = F1 , and X = S1 (1 ), Y = S2 , then


[S1 (1 )S2 | F1 ] = S1 (1 )E
[S2 | F1 ]
E

Albert Cohen (MSU)

MATH 360: Theory of Investment and Credit

MSU Spring 2014

(172)

208 / 223

Taking out what is known: If X is dependent only on the information


contained in G , then
[XY | G ] = X E
[Y | G ]
E

(171)

Example: If G = F1 , and X = S1 (1 ), Y = S2 , then


[S1 (1 )S2 | F1 ] = S1 (1 )E
[S2 | F1 ]
E

(172)

2 | S0 ] ?
Example: What is E[S

Albert Cohen (MSU)

MATH 360: Theory of Investment and Credit

MSU Spring 2014

208 / 223

Taking out what is known: If X is dependent only on the information


contained in G , then
[XY | G ] = X E
[Y | G ]
E

(171)

Example: If G = F1 , and X = S1 (1 ), Y = S2 , then


[S1 (1 )S2 | F1 ] = S1 (1 )E
[S2 | F1 ]
E

(172)

2 | S0 ] ?
Example: What is E[S
Definition: When conditioning on the algebra Fn , we take the
notation
E [X | Fn ] = En [X ]

(173)

This is true, of course, for any measure we use.


Albert Cohen (MSU)

MATH 360: Theory of Investment and Credit

MSU Spring 2014

208 / 223

Martingales

Observe a random process Mn that depends only on the first n coin flips.
The history of the random variable is encapsulated in the filtration Fn it
generates.

Albert Cohen (MSU)

MATH 360: Theory of Investment and Credit

MSU Spring 2014

209 / 223

Martingales

Observe a random process Mn that depends only on the first n coin flips.
The history of the random variable is encapsulated in the filtration Fn it
generates.
If Mn = En [Mn+1 ] then Mn is called a Martingale

Albert Cohen (MSU)

MATH 360: Theory of Investment and Credit

MSU Spring 2014

209 / 223

Martingales

Observe a random process Mn that depends only on the first n coin flips.
The history of the random variable is encapsulated in the filtration Fn it
generates.
If Mn = En [Mn+1 ] then Mn is called a Martingale
If Mn En [Mn+1 ] then Mn is called a Submartingale

Albert Cohen (MSU)

MATH 360: Theory of Investment and Credit

MSU Spring 2014

209 / 223

Martingales

Observe a random process Mn that depends only on the first n coin flips.
The history of the random variable is encapsulated in the filtration Fn it
generates.
If Mn = En [Mn+1 ] then Mn is called a Martingale
If Mn En [Mn+1 ] then Mn is called a Submartingale
If Mn En [Mn+1 ] then Mn is called a Supermartingale

Albert Cohen (MSU)

MATH 360: Theory of Investment and Credit

MSU Spring 2014

209 / 223

Martingales

Observe a random process Mn that depends only on the first n coin flips.
The history of the random variable is encapsulated in the filtration Fn it
generates.
If Mn = En [Mn+1 ] then Mn is called a Martingale
If Mn En [Mn+1 ] then Mn is called a Submartingale
If Mn En [Mn+1 ] then Mn is called a Supermartingale
By the definition and the Tower property above, we have for all k 0

Albert Cohen (MSU)

MATH 360: Theory of Investment and Credit

MSU Spring 2014

209 / 223

Martingales

Observe a random process Mn that depends only on the first n coin flips.
The history of the random variable is encapsulated in the filtration Fn it
generates.
If Mn = En [Mn+1 ] then Mn is called a Martingale
If Mn En [Mn+1 ] then Mn is called a Submartingale
If Mn En [Mn+1 ] then Mn is called a Supermartingale
By the definition and the Tower property above, we have for all k 0
Mn = En [Mn+k ]

(174)

if Mn is a Martingale

Albert Cohen (MSU)

MATH 360: Theory of Investment and Credit

MSU Spring 2014

209 / 223

Martingale Pricing Theorem Under our model with risk neutral


d
probabilities p = 1+r
= u1r
ud , q
ud , the process

Albert Cohen (MSU)

MATH 360: Theory of Investment and Credit

MSU Spring 2014

210 / 223

Martingale Pricing Theorem Under our model with risk neutral


d
probabilities p = 1+r
= u1r
ud , q
ud , the process
Mn :=

Sn
(1 + r )n

(175)

is a martingale

Albert Cohen (MSU)

MATH 360: Theory of Investment and Credit

MSU Spring 2014

210 / 223

Martingale Pricing Theorem Under our model with risk neutral


d
probabilities p = 1+r
= u1r
ud , q
ud , the process
Mn :=

Sn
(1 + r )n

(175)

is a martingale
Proof

Albert Cohen (MSU)

MATH 360: Theory of Investment and Credit

MSU Spring 2014

210 / 223

Martingale Pricing Theorem Under our model with risk neutral


d
probabilities p = 1+r
= u1r
ud , q
ud , the process
Mn :=

Sn
(1 + r )n

is a martingale
Proof
We use the Tower property again:




Sn+1
Sn
1 Sn+1

En
= En
(1 + r )n+1
(1 + r )n 1 + r Sn


Sn
1 Sn+1
=
En
(1 + r )n
1 + r Sn
Sn pu + qd
=
(1 + r )n 1 + r
Sn
=
(1 + r )n
MATH 360: Theory of Investment and Credit
MSU Spring 2014
QEDAlbert Cohen (MSU)

(175)

210 / 223

Let us return to the multiperiod model. For each time n, we will


dynamically redistribute our wealth Xn by deciding to hold n shares of Sn
and invest the rest in the bank account at rate r . At time n + 1, this
means we have the value

Albert Cohen (MSU)

MATH 360: Theory of Investment and Credit

MSU Spring 2014

211 / 223

Let us return to the multiperiod model. For each time n, we will


dynamically redistribute our wealth Xn by deciding to hold n shares of Sn
and invest the rest in the bank account at rate r . At time n + 1, this
means we have the value
Xn+1 = (Xn n Sn ) (1 + r ) + n Sn+1

Albert Cohen (MSU)

MATH 360: Theory of Investment and Credit

MSU Spring 2014

(176)

211 / 223

Let us return to the multiperiod model. For each time n, we will


dynamically redistribute our wealth Xn by deciding to hold n shares of Sn
and invest the rest in the bank account at rate r . At time n + 1, this
means we have the value
Xn+1 = (Xn n Sn ) (1 + r ) + n Sn+1

(176)

If we discount this recursive process to form

Albert Cohen (MSU)

MATH 360: Theory of Investment and Credit

MSU Spring 2014

211 / 223

Let us return to the multiperiod model. For each time n, we will


dynamically redistribute our wealth Xn by deciding to hold n shares of Sn
and invest the rest in the bank account at rate r . At time n + 1, this
means we have the value
Xn+1 = (Xn n Sn ) (1 + r ) + n Sn+1

(176)

If we discount this recursive process to form


Mn :=

Albert Cohen (MSU)

Xn
(1 + r )n

MATH 360: Theory of Investment and Credit

(177)

MSU Spring 2014

211 / 223

Let us return to the multiperiod model. For each time n, we will


dynamically redistribute our wealth Xn by deciding to hold n shares of Sn
and invest the rest in the bank account at rate r . At time n + 1, this
means we have the value
Xn+1 = (Xn n Sn ) (1 + r ) + n Sn+1

(176)

If we discount this recursive process to form


Mn :=

Xn
(1 + r )n

(177)

then Mn is a martingale

Albert Cohen (MSU)

MATH 360: Theory of Investment and Credit

MSU Spring 2014

211 / 223

Risk Neutral Pricing Formula

Assume now that we have the regular assumptions on our coin flip space,
and that at time N we are asked to deliver a path dependent derivative
value VN . Then for times 0 n N, the value of this derivative is
computed via

Albert Cohen (MSU)

MATH 360: Theory of Investment and Credit

MSU Spring 2014

212 / 223

Risk Neutral Pricing Formula

Assume now that we have the regular assumptions on our coin flip space,
and that at time N we are asked to deliver a path dependent derivative
value VN . Then for times 0 n N, the value of this derivative is
computed via


Vn+1

Vn = En
(178)
1+r

Albert Cohen (MSU)

MATH 360: Theory of Investment and Credit

MSU Spring 2014

212 / 223

Risk Neutral Pricing Formula

Assume now that we have the regular assumptions on our coin flip space,
and that at time N we are asked to deliver a path dependent derivative
value VN . Then for times 0 n N, the value of this derivative is
computed via


Vn+1

Vn = En
(178)
1+r
and so

Albert Cohen (MSU)

MATH 360: Theory of Investment and Credit

MSU Spring 2014

212 / 223

Risk Neutral Pricing Formula

Assume now that we have the regular assumptions on our coin flip space,
and that at time N we are asked to deliver a path dependent derivative
value VN . Then for times 0 n N, the value of this derivative is
computed via


Vn+1

Vn = En
(178)
1+r
and so
0
V0 = E

Albert Cohen (MSU)

VN
(1 + r )N

MATH 360: Theory of Investment and Credit

(179)

MSU Spring 2014

212 / 223

Markov Processes
If we use the above approach for a more exotic option, say a lookback
option that pays the maximum over the term of a stock, then we find this
approach lacking. There is not enough information in the tree or the
distinct values for S3 as stated. We need more. Consider our general

multi-period binomial model under P


Definition We say that a process X is adapted if it depends only on the
flips 1 , ..., n
Definition We say that an adapted process X is Markov if for every
0 n N 1 and every function f (x) there exists another function g (x)
such that
n [f (Xn+1 )] = g (Xn )
E

Albert Cohen (MSU)

MATH 360: Theory of Investment and Credit

(180)

MSU Spring 2014

213 / 223

This notion of Markovity is essential to our state-dependent pricing


algorithm. Indeed, since our stock process evolves from time n to time
n + 1, using only the information in Sn , we can in fact say that for every
f (s) there exists a g (s) such that
n [f (Sn+1 ) | Sn = s]
g (s) = E
pf (2s) + qf (0.5s)
g (s) =
1+r

(181)

So, for any f (s) := VN (s), we can work our recursive algorithm backwards
to find the gn (s) := Vn (s) for all 0 n N 1

Albert Cohen (MSU)

MATH 360: Theory of Investment and Credit

MSU Spring 2014

214 / 223

Returning to our example of a lookback option, we see that the problem


was that Mn := max0in Si is not Markov by itself, but the pair (Mn , Sn )
is. Why?
Lets generate the tree!
Homework Can you think of any other processes that are not Markov?
That are Martingales, but not Markov?

Albert Cohen (MSU)

MATH 360: Theory of Investment and Credit

MSU Spring 2014

215 / 223

The Interview Process

Consider the following scenario: After graduating, you go on the job


market, and have 4 possible job interviews with 4 different companies. So
sure of your prospects that you know that each company will make an
offer, with an identically, independently distributed probability attached to
the 4 possible salary offers -

Albert Cohen (MSU)

MATH 360: Theory of Investment and Credit

MSU Spring 2014

216 / 223

The Interview Process

Consider the following scenario: After graduating, you go on the job


market, and have 4 possible job interviews with 4 different companies. So
sure of your prospects that you know that each company will make an
offer, with an identically, independently distributed probability attached to
the 4 possible salary offers P [Salary Offer=x1 ] = p1
P [Salary Offer=x2 ] = p2
P [Salary Offer=x3 ] = p3

(182)

P [Salary Offer=x4 ] = p4
p1 + p2 + p3 + p4 = 1

Albert Cohen (MSU)

MATH 360: Theory of Investment and Credit

MSU Spring 2014

216 / 223

The Interview Process

How should you interview?

Albert Cohen (MSU)

MATH 360: Theory of Investment and Credit

MSU Spring 2014

217 / 223

The Interview Process

How should you interview?


Specifically, when should you accept an offer and cancel the
remaining interviews?

Albert Cohen (MSU)

MATH 360: Theory of Investment and Credit

MSU Spring 2014

217 / 223

The Interview Process

How should you interview?


Specifically, when should you accept an offer and cancel the
remaining interviews?
How does your strategy change if you can interview as many times as
you like, but the distribution of offers remains the same as above?

Albert Cohen (MSU)

MATH 360: Theory of Investment and Credit

MSU Spring 2014

217 / 223

Lets review the basic contracts we can write:

Albert Cohen (MSU)

MATH 360: Theory of Investment and Credit

MSU Spring 2014

218 / 223

Lets review the basic contracts we can write:


Forward Contract Initial Value is 0, because both buyer and seller
may have to pay a balance at maturity

Albert Cohen (MSU)

MATH 360: Theory of Investment and Credit

MSU Spring 2014

218 / 223

Lets review the basic contracts we can write:


Forward Contract Initial Value is 0, because both buyer and seller
may have to pay a balance at maturity
(European) Put/Call Option Initial Value is > 0, because both only
seller must pay balance at maturity.

Albert Cohen (MSU)

MATH 360: Theory of Investment and Credit

MSU Spring 2014

218 / 223

Lets review the basic contracts we can write:


Forward Contract Initial Value is 0, because both buyer and seller
may have to pay a balance at maturity
(European) Put/Call Option Initial Value is > 0, because both only
seller must pay balance at maturity.
(European) Exotic Option Initial Value is > 0, because both only
seller must pay balance at maturity.

Albert Cohen (MSU)

MATH 360: Theory of Investment and Credit

MSU Spring 2014

218 / 223

Lets review the basic contracts we can write:


Forward Contract Initial Value is 0, because both buyer and seller
may have to pay a balance at maturity
(European) Put/Call Option Initial Value is > 0, because both only
seller must pay balance at maturity.
(European) Exotic Option Initial Value is > 0, because both only
seller must pay balance at maturity.
During the term of the contract, can the value of the contract ever fall
below the intrinsic value of the payoff? Symbolically, does it ever occur
that

Albert Cohen (MSU)

MATH 360: Theory of Investment and Credit

MSU Spring 2014

218 / 223

Lets review the basic contracts we can write:


Forward Contract Initial Value is 0, because both buyer and seller
may have to pay a balance at maturity
(European) Put/Call Option Initial Value is > 0, because both only
seller must pay balance at maturity.
(European) Exotic Option Initial Value is > 0, because both only
seller must pay balance at maturity.
During the term of the contract, can the value of the contract ever fall
below the intrinsic value of the payoff? Symbolically, does it ever occur
that
vn (s) < g (s)

Albert Cohen (MSU)

MATH 360: Theory of Investment and Credit

(183)

MSU Spring 2014

218 / 223

Lets review the basic contracts we can write:


Forward Contract Initial Value is 0, because both buyer and seller
may have to pay a balance at maturity
(European) Put/Call Option Initial Value is > 0, because both only
seller must pay balance at maturity.
(European) Exotic Option Initial Value is > 0, because both only
seller must pay balance at maturity.
During the term of the contract, can the value of the contract ever fall
below the intrinsic value of the payoff? Symbolically, does it ever occur
that
vn (s) < g (s)

(183)

where g (s) is of the form of g (S) := max {S K , 0}, in the case of a Call
option, for example.
Albert Cohen (MSU)

MATH 360: Theory of Investment and Credit

MSU Spring 2014

218 / 223

For Freedom! (we must charge extra...)

What happens if we write a contract that allows the purchaser to exercise


the contract whenever she feels it to be in her advantage? By allowing this
extra freedom, we must

Albert Cohen (MSU)

MATH 360: Theory of Investment and Credit

MSU Spring 2014

219 / 223

For Freedom! (we must charge extra...)

What happens if we write a contract that allows the purchaser to exercise


the contract whenever she feels it to be in her advantage? By allowing this
extra freedom, we must
Charge more than we would for a European contract that is exercised
only at the term N

Albert Cohen (MSU)

MATH 360: Theory of Investment and Credit

MSU Spring 2014

219 / 223

For Freedom! (we must charge extra...)

What happens if we write a contract that allows the purchaser to exercise


the contract whenever she feels it to be in her advantage? By allowing this
extra freedom, we must
Charge more than we would for a European contract that is exercised
only at the term N
Hedge our replicating strategy X differently, to allow for the
possibility of early exercise

Albert Cohen (MSU)

MATH 360: Theory of Investment and Credit

MSU Spring 2014

219 / 223

For Freedom! (we must charge extra...)

What happens if we write a contract that allows the purchaser to exercise


the contract whenever she feels it to be in her advantage? By allowing this
extra freedom, we must
Charge more than we would for a European contract that is exercised
only at the term N
Hedge our replicating strategy X differently, to allow for the
possibility of early exercise
Extend the notion of the replicating strategy as a Martingale, when
properly discounted

Albert Cohen (MSU)

MATH 360: Theory of Investment and Credit

MSU Spring 2014

219 / 223

Some examples:
American Bond: g (s) = 1
American Digital Option: g (s) = 1{6s10} and
1
= q
2
1
r=
4
p =

S0 = 4, u = 2, d =

Albert Cohen (MSU)

(184)
1
2

MATH 360: Theory of Investment and Credit

MSU Spring 2014

220 / 223

Using the state-variable version of our Risk-Neutral pricing algorithm, we


were able to compute the fair price of the European option as V0 .

Albert Cohen (MSU)

MATH 360: Theory of Investment and Credit

MSU Spring 2014

221 / 223

Using the state-variable version of our Risk-Neutral pricing algorithm, we


were able to compute the fair price of the European option as V0 .
How would we compute this V0 for the corresponding American
option?
What would our tree look like?
What would our strategy be?

Albert Cohen (MSU)

MATH 360: Theory of Investment and Credit

MSU Spring 2014

221 / 223

Using the state-variable version of our Risk-Neutral pricing algorithm, we


were able to compute the fair price of the European option as V0 .
How would we compute this V0 for the corresponding American
option?
What would our tree look like?
What would our strategy be?
Certainly, for any strategy X we enact, we must have
Xn max {K Sn , 0}

(185)

to reflect the early payoff the option holder can take.

Albert Cohen (MSU)

MATH 360: Theory of Investment and Credit

MSU Spring 2014

221 / 223

Stopping Times

The question of when exactly an investor will choose to prune the tree
and take her payoff must be asked. In precise language, we define a
Stopping Time as an adapted random variable on our discrete
probability space (, F, P) with filtration {Fk }N
k=0
: {0, 1, 2, ...N}
{ | () = k} Fk k = 0, 1, 2, .., N

Albert Cohen (MSU)

MATH 360: Theory of Investment and Credit

MSU Spring 2014

(186)

222 / 223

Stopping Times:Example
Consider the case of
1
= q
2
1
r=
4

p =

(187)

1
S0 = 4, u = 2, d = , N = 2
2
V2 := max {K S2 , 0}
Then for := min {m | vm (Sm ) = {K Sm , 0}}, we have
{ | () = 0} = F0
{ | () = 1} = {TH, TT } F1

(188)

{ | () = 2} = {HH, HT } F2
and so is a stopping time. Can you come up with a random time that is
not a stopping time?
Albert Cohen (MSU)

MATH 360: Theory of Investment and Credit

MSU Spring 2014

223 / 223

You might also like